Jump to content

Wikipedia:Reference desk/Science: Difference between revisions

From Wikipedia, the free encyclopedia
Content deleted Content added
Line 693: Line 693:


:: I suspect the original poster means [[virtual work]]. -- [[User:Finlay McWalter|Finlay McWalter]]'''ჷ'''[[User talk:Finlay McWalter|Talk]] 18:25, 6 May 2012 (UTC)
:: I suspect the original poster means [[virtual work]]. -- [[User:Finlay McWalter|Finlay McWalter]]'''ჷ'''[[User talk:Finlay McWalter|Talk]] 18:25, 6 May 2012 (UTC)

== Is it safe to keep batteries in a tin can? ==

I keep my batteries in a red tin can and a brown tin can. I put the dead ones in the red can and the ones that I haven't used yet in the brown can. After a while this chemical smell started come around. So is this safe? Oh and can I recycle batteries? Matthew Goldsmith 18:58, 6 May 2012 (UTC)

Revision as of 18:58, 6 May 2012

Welcome to the science section
of the Wikipedia reference desk.
Select a section:
Want a faster answer?

Main page: Help searching Wikipedia

   

How can I get my question answered?

  • Select the section of the desk that best fits the general topic of your question (see the navigation column to the right).
  • Post your question to only one section, providing a short header that gives the topic of your question.
  • Type '~~~~' (that is, four tilde characters) at the end – this signs and dates your contribution so we know who wrote what and when.
  • Don't post personal contact information – it will be removed. Any answers will be provided here.
  • Please be as specific as possible, and include all relevant context – the usefulness of answers may depend on the context.
  • Note:
    • We don't answer (and may remove) questions that require medical diagnosis or legal advice.
    • We don't answer requests for opinions, predictions or debate.
    • We don't do your homework for you, though we'll help you past the stuck point.
    • We don't conduct original research or provide a free source of ideas, but we'll help you find information you need.



How do I answer a question?

Main page: Wikipedia:Reference desk/Guidelines

  • The best answers address the question directly, and back up facts with wikilinks and links to sources. Do not edit others' comments and do not give any medical or legal advice.
See also:


April 29

How do I mimic static over the cellphone?

In some situations, I feel the need to mimic increasing static on a phone before disconnecting. It needs to sound convincing, so what are the best methods? Thanks. --68.102.29.129 (talk) 02:26, 29 April 2012 (UTC)[reply]

Stand outside, hold phone into wind. --Mr.98 (talk) 02:53, 29 April 2012 (UTC)[reply]
This isn't always convenient when the day is calm, and when I'm in a bad position to go outside and hold the phone into the wind. Are there other methods just as good? --68.102.29.129 (talk) 03:36, 29 April 2012 (UTC)[reply]
Modern digital communication rarely injects Gaussian noise into the audio channel. A dropped call often occurs without warning, and the call simply disconnects. If you're seeking to realistically simulate a call interrupted by poor reception, simply disconnect at a random time. If you actually want to play "static" over the microphone for some other reason, consider recording some white noise and playing it in the background using an audio player. Nimur (talk) 04:10, 29 April 2012 (UTC)[reply]
Well, there is comfort noise. DMacks (talk) 09:01, 29 April 2012 (UTC)[reply]
I don't know about cell phones in particular, but in general simply blowing directly onto a microphone tends to produce a surprising amount of noise, unless you're using a dead cat or something. Red Act (talk) 04:40, 29 April 2012 (UTC)[reply]
The best trick might be to hang up while you yourself are talking, reducing the possibility that the other party will think you hung up on them. ←Baseball Bugs What's up, Doc? carrots04:52, 29 April 2012 (UTC)[reply]
I agree - digital cellphones don't often get noisy before dropping out, but what they often do is chop up / interrupt your voice a few times before disconnecting. You could simulate this by rapidly placing and removing your finger over the microphone hole while continuing your talking. On most modern phones, it's only a single hole 0.5 to 1 mm diameter. If you do this more than once or twice to the same caller, they'll will catch on to what you are doing though. Wickwack121.221.228.248 (talk) 05:34, 29 April 2012 (UTC)[reply]

A Fibres in Frog Sciatic Nerve

Hello. Are the A-alpha fibres of the frog sciatic nerve the only component of A fibres that are myelinated? Thanks in advance. --Mayfare (talk) 18:20, 29 April 2012 (UTC)[reply]

I don't know the answer but a google snippet suggested that this paper might have some information. SpinningSpark 10:54, 30 April 2012 (UTC)[reply]

Gravity

Physicists variously talk about gravity as an element of space-time, as gravity waves, and as gravitons. Are these related in some way or are they three different views of gravity? — Preceding unsigned comment added by Gstrom47 (talkcontribs) 20:19, 29 April 2012 (UTC)[reply]

Gravitational waves and viewing gravity as the curvature of spacetime are part of the same model of gravity, general relativity. Gravitons are an attempt to extend quantum field theory to gravity, which is a different approach in that general relativity doesn't deal with quantum mechanics at all. Red Act (talk) 21:02, 29 April 2012 (UTC)[reply]
On the other hand, I don't want to give the impression that general relativity and gravitons are completely unrelated. Two ways in which gravitational waves and gravitons are related are that both travel at the speed of light, and they both have the same symmetry: if you rotate either a gravitational wave or a graviton by 180 degrees, it will wind up looking the same as before the rotation. It's just that one involves a quantum field and the other involves a classical (nonquantum) field, very similar to the relationship between electromagnetic waves and photons. Red Act (talk) 21:53, 29 April 2012 (UTC)[reply]
They are all closely related. A graviton is a quantum of gravitational wave energy, and Feynman diagrams with virtual gravitons are mathematically related to the field equations of general relativity in much the same way that a Taylor series is related to the function it approximates. -- BenRG (talk) 00:48, 30 April 2012 (UTC)[reply]
Agree with all of the above. General relativity is a classical field theory and so gives rise to gravitational waves which are analagous to electromagnetic waves in classical electromagnetism. If you try to develop a quantum field theory version of general relativity (which is one of the possible routes to a quantum theory of gravity) then gravitational waves are replaced by gravitons, in the same way as photons replace electromagnetic waves in quantum electrodynamics. So gravitational waves and gravitons are two different ways of describing the same phenomena - one classical, one quantum. However, in contrast to electromagnetism, we have not yet directly detected gravitation waves (although there is some indirect evidence for their existence) or gravitons. Gandalf61 (talk) 10:53, 30 April 2012 (UTC)[reply]
Both you and Red Act suggest that gravitons replace gravitational waves in quantum field theories. That seems very misleading to me. "Graviton" is just another word for "gravitational wave" in quantum field theory. It's equally true that in quantum theory sound is replaced by phonons. Anything involving vibration gets quantized. That's no call to invent new terms for everything, much less to treat them as new concepts. -- BenRG (talk) 17:29, 30 April 2012 (UTC)[reply]
Your QFT knowledge is much stronger than mine, but my understanding is that assuming that gravitons exist, then gravitational waves of the strength for example being searched for by LIGO et al can basically be viewed as a coherent state of many gravitons. However, although the existence of gravitons would imply the existence of gravitational waves, the converse is not true; gravitational waves are predicted to exist (by GR) independently of whether gravity actually quantizes into gravitons. To put it another way, if LIGO and even more powerful detectors continue to fail to find gravitational waves, that would be a blow to GR in a way that, say, a theoretical result in QFT showing that gravitons can't actually exist would not be. So gravitational waves and gravitons are at least different in that regard. Would you agree with that assessment? Red Act (talk) 18:48, 30 April 2012 (UTC)[reply]
Actually, I stand by "replaced". A classical wave is a solution to a system of PDEs such as the wave equation, Maxwell's equations or the Einstein field equations. It assumes that space, time and other properties of the system such as energy and momenta are continuous and take on definite values, and our knowledge of their values is limited only by the sensitivity of our instruments. A quantum model is fundamentally different. It involves probability waves that are superficially similar to the waves in the classical model, but we cannot observe them directly due to wave function collapse. What we observe are quanta. Only when a large number of quanta are observed does the classical wave picture emerge at a macroscopic level. Gandalf61 (talk) 14:05, 1 May 2012 (UTC)[reply]

Why were the X-15 rocket-powered aircrafts retired?

Why were the X-15 rocket-powered aircrafts retired? 82.31.133.165 (talk) 21:09, 29 April 2012 (UTC)[reply]

As the X indicates, they were experimental aircraft, never intended to be useful for any practical purpose. They were intended to test a concept, and once that concept was proven, there was no longer much point in flying them. Looie496 (talk) 21:19, 29 April 2012 (UTC)[reply]
Indeed. Compared with other experimental aircraft, X-15 had a remarkably long career (over 10 years, 199 flights, on three airframes). X-15 research fed directly into the space shuttle's glide phase design (and I imagine into classified hypersonic control projects like MIRVs). -- Finlay McWalterTalk 21:24, 29 April 2012 (UTC)[reply]
As for rocket-powered aircraft, in general, they suffer from a rather short flight time, which makes them less flexible (you can't circle the runway for long, waiting for a chance to land). A large turn radius is another problem. StuRat (talk) 21:31, 29 April 2012 (UTC)[reply]
The X-15 was a military research vehicle. The USAF's plan was to incrementally develop space transport - there should have been a successor to the X-15. However, the response of the US Govt to the success of Sputnik and other Russian space projects was to decide that a) a "space race" was necessary, and the USA must win it, and b) it must be done with a civilian effort. This was so that that orbiting altitudes and space did not becaome a military domain. Thus the military programme was terminated, and NASA created, with von Braun in charge. von Braun was partial to large "brute force" vertically launched disposable rockets, rather than re-usable flyable planes. Ratbone58.170.164.197 (talk) 14:58, 30 April 2012 (UTC)[reply]
That makes for a nice story, but the first X-15 flights didn't occur until after NASA was created, and the last of the 199 flights was more than a decade after Sputnik started the space race. Of the 12 people to ever fly an X-15, five were NASA personnel, including the first man on the moon Neil Armstrong. Yes, the successor vehicle was terminated in favor of the heavy lift rockets that came to represent the space race; however, it is too simplistic to say that X-15 was killed simply because it was military. The X-15 was originally proposed and created as a joint operation between the USAF and the civilian NACA program, which later became NASA. As a joint USAF / NASA project, it enjoyed a far longer life than most X- series experimental flight vehicles. Dragons flight (talk) 15:25, 30 April 2012 (UTC)[reply]
Nowhere did I say the X-15 was killed off - I only said the military programme (ie X15B, Dynasoar etc) was terminated and there was no successor when there should have been. It is misleading to say the X15 did not fly until NASA was created, without further qualification. It takes much much longer to design and build a air/space vehicle than it does to create a govt agency. Planning for the X-15 goes back to at least 1955, probably even further in meetings, discussions and the like not in the public domain. Nobody thought of creating NASA until 1958. Pres Eisenhower did not sign the enabling law into effect until late 1958, and Russian space achievements were the driver. Ratbone124.182.4.180 (talk) 02:55, 1 May 2012 (UTC)[reply]

Preventing water damage when building a building

Is it necessary, since the building still doesn't have roof, or is it not, since it's just concrete? XPPaul (talk) 22:12, 29 April 2012 (UTC)[reply]

If the builders get to the point where there are things inside the building that would be damaged by water before the roof is finished, then can cover the top with plastic sheeting or something similar (I have seen that on building sites). I think it's normal to finish the roof before you start to work on the interior, though, so it would just be concrete and bricks - a bigger problem is if you are working on the roof of an existing building, then you some kind of temporary protection. --Tango (talk) 22:34, 29 April 2012 (UTC)[reply]
Just to make things clear, a bare reinforced concrete frame of a building in construction won't get damaged by direct water contact?XPPaul (talk) 23:09, 29 April 2012 (UTC)[reply]
Well, the concrete needs a dry period to set. After that, the concrete itself should be OK, but if there's no drainage, water could accumulate and undermine the foundation, causing it to fail. Rebar can also rust, if sitting in water. This could also cause the concrete to fail. I seem to recall a walking bridge over a highway failing that way a few years back. Then filler in low-quality concrete may also be damaged by water. Chunks of iron, for example, will also rust there. Most of these effects would require a lot of time and water, though, so one or two rainstorms won't be a problem. StuRat (talk) 23:51, 29 April 2012 (UTC)[reply]
There can be a problem with timber that has too much moisture if it is going to be enclosed and unable to dry out. SpinningSpark 23:23, 29 April 2012 (UTC)[reply]

Niagara falls into dam

Is it theoretically possible that the Niagara Falls could be turned into a hydroelectric dam? Also, how long will it take until the Great Lakes drain into Lake Ontario? 64.229.204.143 (talk) 22:46, 29 April 2012 (UTC)[reply]

Anywhere with a significant flow of water could be turned into a hydroelectric generator. They don't usually use waterfalls, though - it's easier to create your own drop using a dam. There would be a lot of engineering problems with using the Niagara falls, and they would also lose all the tourism. I'm not sure what you mean about the Great Lakes - they aren't draining out, there is always water being added from rivers that drain into them. --Tango (talk) 23:04, 29 April 2012 (UTC)[reply]
I don't think a flow of water is sufficient. You also need a canyon, so your dam can be like the plug in a bathtub. To build a dam on flat ground would require that you build the entire "bathtub", which would cost more than it would be worth. In the case of Niagara Falls, I believe the part above the falls is fairly flat. StuRat (talk) 23:42, 29 April 2012 (UTC)[reply]
No canyon necessary, but a slow flow (at low pressure) would not generate electricity very efficiently, so a reasonable height difference is usually preferred. The advantage of using a canyon (or similar geographical feature) is that a large reservoir can be maintained at reasonable cost, so that generation can continue in dry weather. Dbfirs 06:52, 30 April 2012 (UTC)[reply]
If you mean no canyon is necessary for hydroelectric power generation, I agree, since this is currently done at Niagara. However, if you mean that a large dam can be constructed economically without a canyon, which is what this question is about, then I disagree. StuRat (talk) 15:13, 30 April 2012 (UTC)[reply]
No, I wasn't suggesting that a large dam would be economic without some sort of natural valley, so we are in agreement, except on what the question is about. My point was that only a height difference is important, not a dam or canyon. Dbfirs 07:25, 1 May 2012 (UTC)[reply]
The title and first sentence would seem to indicate that the question was about a dam, not other forms of hydroelectric generation. StuRat (talk) 07:30, 1 May 2012 (UTC) [reply]
... but it doesn't say "large dam". Anyway, we agree on the essentials, and that generation is separate from damming. Dbfirs 07:10, 2 May 2012 (UTC)[reply]
The land around Niagara Falls is pretty flat, although there are escarpments, like the Niagara Escarpment, which is why there's a waterfall there at all. Also there is a canyon, or at least a gorge, below the falls, extending to about Lewiston, NY (Google Maps terrain shows it here, [1]). It would probably be possible, in theory anyway, to build a dam near Lewiston, creating a reservoir backing up to the falls. It's easy to think of various reasons why this hasn't been done, and won't be. Instead they built "off-river" reservoirs. Pfly (talk) 01:33, 1 May 2012 (UTC)[reply]
Also, for more on hydroelectric dams with minimal reservoirs, see Run-of-the-river hydroelectricity. Pfly (talk) 01:41, 1 May 2012 (UTC)[reply]
One final tidbit I can't help but mention--there have been some very grand plans envisioned for Niagara Falls. One of the wildest was described in The Human Drift by King C. Gillette--the guy who founded the Gillette company. He pictured a giant utopian city built right on top of Niagara Falls, rectangular in shape, 135x45 miles large, built out of porcelain, the only city on the entire continent, powered with electricity from the falls, and run by a "world corporation". Pfly (talk) 01:56, 1 May 2012 (UTC)[reply]
Kinda like Fallingwater, only much bigger. ←Baseball Bugs What's up, Doc? carrots12:03, 2 May 2012 (UTC)[reply]
Niagra Falls is used for hydroelectric power generation - see Niagra Falls#Power. So much so that the amount of water the power plant uses is limited by treaty for fear of the Falls drying up to a trickle. SpinningSpark 23:28, 29 April 2012 (UTC)[reply]
According to our Niagara River page it was the very first river used for hydroelectricity in North America. There's a monument to Nikola Tesla on Goat Island right at the falls. In satellite imagery, like File:Niagara Falls SPOT 1314.jpg, you can see the two large reservoirs used for hydroelectricity generation (one in the US and one in Canada). Water is diverted from the river above the falls into the two reservoirs and then returned to the river below the falls, falling through large turbines to make electricity. In short, not only is Niagara Falls already used for making electricity, it is one of the most famous and historic examples of hydroelectric power. Pfly (talk) 23:36, 29 April 2012 (UTC)[reply]
As has already been noted, the Great Lakes are continually refilled with rain and snow melt. However, even if all precipitation in the Great Lakes watershed ended, the lakes still wouldn't "drain", as the lakes are deeper than the rivers connecting them (Detroit River, Saint Clair River, Niagara River, Saint Lawrence River). So, each lake would only drain to the depth of it's outlet river. From there, it would eventually evaporate. StuRat (talk) 15:19, 30 April 2012 (UTC)[reply]
(psst, also the St. Marys River!) Pfly (talk) 04:15, 1 May 2012 (UTC)[reply]
Thanks, couldn't remember the name of that one. There's also the Straits of Mackinac, although I'm not sure if that's shallower than Lake Michigan. StuRat (talk) 08:40, 1 May 2012 (UTC)[reply]
Also don't forget the Chicago River which was canalized from an inlet river to an outlet river. It is technically feasible to get from New Orleans to the mouth of the St. Lawrence via boat. Another way to put that is that the entire eastern portion of the U.S. is technically an island, as it is completely surrounded by water (though not all natural sources). --Jayron32 23:20, 1 May 2012 (UTC)[reply]
Kinda like Cape Cod, only much bigger. ←Baseball Bugs What's up, Doc? carrots12:04, 2 May 2012 (UTC)[reply]

What are typical Power capacities for suburban Distribution Substations??98.225.64.151 (talk) 23:13, 29 April 2012 (UTC)

I live in Arizona, with major summertime Air-Con loads. I do know that my own neighborhood substation was built in 1985 on about 1 acre of land, and is fed by a 138kV line. I've asked my Utility for that substation's Power rating, but they won't tell me because of "security reasons." Thanks in advance to you Power engineers.

You are asking the wrong question and/or are asking a question, which if you had the answer, the answer would have no meaning for you. 138kV lines would generally be used to supply power at levels exceeding 20 MW (megawatt), up to 600 MW or more. This is a power level that is appropriate for entire districts up to significant cities. A substation requiring 1 acre is a huge substation. If you want to know if what feeds your house is adequate, you need to focus on the distrubtion transformer and/or RMU (Ring Main Unit, a type of switch) that feeds your street. This is typically about the size of one or two domestic fridges. In most countries, these are mounted up on poles if you have open wire distribution, or in roadside cabinets in underground cabling is used. In the USA, with undertground street cabling, the transformers & RMU's are often underground as well. Instead of asking what is the power rating, you coukld ask something like what is the site design power capacity, and what has been the load growth over (say) the last 10 years. But even this will be pretty meaningless, because there is a lot of substation reconfiguration, load transfer, and other upgrade work the power company can do, so that even if a substation appears to be at 100% capacity, there is likley nothing electricity consumers should be concerned about. In any case, substations at the level you are talking about are usually fully redundant - if there is a fault, the power company can changing switching and feed the loads from somethwere else, and they can switch loads as required to even up the loads on various interconnected substations. Wickwack121.221.89.56 (talk) 03:08, 30 April 2012 (UTC)[reply]

It is a very reasonable and sensible question, but not one easily answered based only on the area of the substation and the incoming transmission voltage. 138kv lines differ in their current capabilities, based on conductor size, for instance. To figure out the answer to your question, one would need to know the transmission capability of the 138kv line or lines entering the substation, and the capabilities of the power system (generating stations or substations) supplying those 138kv lines. That represents the maximum power could be supplied to transformers at the distribution substation. The next questions is how many transformers of what megawatt rating are located at the distribution substation, to step the 138kv transmission down to 34kv, 12 kv or 4kv distribution. Other elements which could limit the power are any other things the current might pass through, such as current transformers, inductors, bus bars, switches, circuit breakers and wavetraps. A utility strives to make sure that some cheap element is not the limiting factor. Then one could consider the capabilities of the distribution lines leaving the substation. In the end, a maximum power capability could be computed, based not on the strongest but on the weakest link in the chain. One utility I am familiar with can have up to four 40 MVA (megavoltamp) transformers stepping down 138kv transmission from two incoming transmission lines to about 30 12 kv distribution lines, for a total of 160 MVA. I know of other substations in urban areas with four 50 mva transformers. Other utilities might have varying ultimate designs for their substations. (The megawatts would be somewhat less than the MVA due to the power factor of motors etc being less than one). A utility might start with one 30 mva 138kv-12kv transformer transformer in a substation. Edison (talk) 01:50, 1 May 2012 (UTC)[reply]
I agree with Wickwack - the "power rating" is not a question meaningfull or relavent to a consumer. However the rest of what Edison says is valid and supports Wickwack. It should also be noted that the effective power (or (VA) capacity of a substation may depend on its' switched configuration: E.g., due to internal bus limitations, if power is drawn from incomming feeds A and B and switched to supply distributions X and Y, while feed C supplies distribution W, the total capacity might be different if say distribution B is switched on to feed D. Particular distributions may be feed from more than one substation. Sometimes large consumers with critical functions (eg satelite tracking stations, hospitals) have feeders from two or more substations so that they have power no matter what accident or fault happens in either feeders or substations. Ratbone124.182.4.180 (talk) 03:10, 1 May 2012 (UTC)[reply]
Also, a utility typically builds in a LOT of redundancy, so that one or more transmission lines, transformers, bus ties, bus sections, and distribution lines can be out of service without having to drop load. They are not so optimistic as to assume everything will be working perfectly on a peak load day. A substation might be expected to serve only half the load it is nominally capable of, to allow for contingencies. Additionally, the "stiffness" or available fault current from a substation might have an impact on the amount of flicker all the other customers from a substation see when some industrial load such as a steelmaking arc furnace, or a very large motor is switched on and off frequently. This is one aspect of the power available besides the issue of how much current it could supply without burning something out (transmission line, transformer, etc). Edison (talk) 03:33, 1 May 2012 (UTC)[reply]

Leftover rabies?

So it seems 2 animals of some sort had a fight in my back yard last night. Bits of fur and whatnot laying about. So: IF one or more had rabies, AND IF said animal bled or drooled or whatnot on the ground, what danger would there be of being exposed by being in that area? Obviously I didn't roll around in the grass, but I'm just curious. Thanks in advance. — Preceding unsigned comment added by 71.62.103.2 (talk) 23:31, 29 April 2012 (UTC)[reply]

Pretty low, unless you intend to use the drool-covered fur to make a bandage for a cut on your arm. Even then you probably wouldn't get rabies (but might get some other infection). StuRat (talk) 23:39, 29 April 2012 (UTC)[reply]
This website has a fairly detailed description of what counts as exposure, and surprisingly (to me at least) it states that contact with blood of an infected animal does not count as exposure. Of course in your case there might be some saliva mixed in with the blood, so I still wouldn't go touching it. Vespine (talk) 23:54, 29 April 2012 (UTC)[reply]
I don't believe you can get rabbies by touching contaminated saliva or blood. Bites are obviously a different ball park. XPPaul (talk) 00:11, 30 April 2012 (UTC)[reply]
Actually you can should you have exposed wounds or get it on mucous membranes. See this from the CDC which gives nonbite criteria. Our article, Rabies transmission, mentions cave explorers being infected, although TTBOMK the route of transmission such as aerosols IMO was somewhat speculative although not unreasonable. To safeguard other scavenging animals such as pets from getting into the mess, I'd use a shovel to remove any large pieces and rinse the area down with a garden hose. Plus, avoid not tracking any of it in with your shoes. --Modocc (talk) 01:05, 30 April 2012 (UTC)[reply]
Yes the website I linked does explain that it is possible to be infected through "non-bite exposure" to saliva or CNS tissues, such as brain matter. As the above states, you would need to contact mucous membrane (which can be as simple as rubbing your eye) or a wound such as sa cratch or abrasion. Vespine (talk) 01:28, 30 April 2012 (UTC)[reply]


April 30

Fibrosis

Do all infections and inflammations cause scar tissue to form in the area affected? — Preceding unsigned comment added by 138.253.210.6 (talk) 08:17, 30 April 2012 (UTC)[reply]

I don't think so otherwise the common cold would scar nasal tissue, and that's obviously not true. I stand to be corrected though. --TammyMoet (talk) 09:30, 30 April 2012 (UTC)[reply]
No. Small areas of damage to the body are fixed by regrowing the original tissue, with a few exceptions, like nerve cells. Thus, no damage should be apparent once this process is complete. However, this is too slow for large areas of damage, so scar tissue is used, instead. Unfortunately, we lack the ability to gradually replace scar tissue with normal tissue, so scars become permanent. StuRat (talk) 15:30, 30 April 2012 (UTC)[reply]

Kinased oligonucleotides

Hello. What's a "kinased oligonucleotide", and how does it differ from a "normal" oligonucleotide? Thanks! Leptictidium (mt) 10:25, 30 April 2012 (UTC)[reply]

Does protein kinase have the information you need? SpinningSpark 15:48, 30 April 2012 (UTC)[reply]
Kinases are enzymes that facilitate the addition of phosphate groups to other molecules; a 'kinased' oligonucleotide is therefore one to which a phosphate has been added (generally to the 5' end) through the use of an enzyme (most often T4 or T7 polynucleotide kinase). TenOfAllTrades(talk) 15:49, 30 April 2012 (UTC)[reply]
Why it's not just called a phosphorylated oligo is beyond me..... Fgf10 (talk) 17:01, 30 April 2012 (UTC)[reply]
Probably historical accident. I'm guessing that the T4 PNK method was either the first, the cheapest, or the most popular method for some period of time (and may still be—I've never done much work in the area) and it has just stuck. The term phosphorylated is definitely also used; it is slightly broader in its meaning, however, in that it can also cover oligos that were phosphorylated using other mechanisms (that is, not with a kinase) or at the other (3') end of the strand. If it makes you feel better, Google suggests that phosphorylated oliogos outnumber their kinased brethren on the web by about three to one. TenOfAllTrades(talk) 13:12, 1 May 2012 (UTC)[reply]
Think of it from the worker's point of view. He has these oligos, he has to "kinase" them, and then he has "kinased oligos". Wnt (talk) 05:11, 3 May 2012 (UTC)[reply]

How many neurons can fire at once?

I've read the average neuron in the human brain can fire 200 times per second. How many neurons can actually sustain this rate simultaneously? How many usually do? Is this limited by physical constraints (e.g. ability to feed in and dissipate energy) or by neurological bottlenecks? NeonMerlin 12:11, 30 April 2012 (UTC)[reply]

I suspect that while the (limitted) supply of nutrients will enforce an upper limit of how many many neurons can simulataneously fire at maxiumum rate, electric disturbance will impose a lower limit. Every neuron that fires creates an electric field that propagates out through the bulk of the brain. If you get enough firing simultaneously and rythmically, the electric fields will add, perhaps creating a field strong enough to fire more neurons. This could avalanche thru the brain causing loss of functioning until nutrients and/or neurotransmitters are finally expended. Any comments from those who have studied epilepsy? Ratbone58.170.164.197 (talk) 14:49, 30 April 2012 (UTC)[reply]
the high Amplitude of Delta wave production indicates synchronized firing of many neurons (at low frequency). I believe Exploding head syndrome and Life review are other examples of simultaneous activity of large percentage of neurons. --Digrpat (talk) 22:00, 30 April 2012 (UTC)[reply]
Does this mean that EHS or seizures could (at least in theory) result if the executive functions were somehow artificially accelerated too much? (The idea of such acceleration was what motivated the question, because I've read about the "executive bottleneck" hypothesis and am interested in cognitive enhancement.) NeonMerlin 12:17, 1 May 2012 (UTC)[reply]

200 Hz is a rate you only see in brief bursts, usually not more than 5 or 10 spikes. There are areas of the brain where most of the neurons run constantly at 50 Hz or more, but those areas only make up a small fraction of the total brain. In the cerebral cortex, the great majority of neurons fire at overall time-averaged rates of 10 Hz or less, although individual neurons can ramp up quite a bit from that for brief periods when they are active. During an epileptic seizure the rates go up substantially, but I don't have any figures -- I'm pretty sure the max rate is far below 200 Hz, though. Looie496 (talk) 04:20, 1 May 2012 (UTC)[reply]

Hydraulics/Pressure question

I have a box with a shaft passing through it. The box (completely insulated of course) is filled with a hydraulic fluid maintained at a certain pressure (say, 10 bar). When the shaft rotates (say, at 1500 RPM), will the pressure acting on the walls of the box change? If so, how? 117.216.154.47 (talk) 12:39, 30 April 2012 (UTC)[reply]

Somewhat irrespective of the pressure within the box, as any practical fluid will have a finite viscosity, the rotating shaft will tend to drag round the fluid with it, the the thereby rotating fluid will try to drag round the box as well. If the box is not externally constrained, then it will rotate at the same speed as the shaft. If the box is externally prevented from rotating, then there will be power dissipated in the fluid. If viscosity is sufficient, the power absorbed in the fluid will raise its' temperature. All practical fluids expand slightly with temperature, so if the box is trulely rigid, and the seals perfect, infinite pressure could in theory be attained. In practice, at reasonable speeds like 1500 RPM, things will be different. Practical fluids show a marked decrease in viscosity with temperature, and (usually) a very slight decrease in viscosity with pressure. This means that in practice, you are likely to have the fluid temperature settle at a slightly raised temperature with little change in pressure (depending on the box & shaft dimensions, perhaps not even measurable), as it will settle at a temperature just great enough to reduce the viscosity so as to absorb just enough power to maintain that temperature. See http://en.wikipedia.org/wiki/Viscosity Ratbone58.170.164.197 (talk) 14:38, 30 April 2012 (UTC)[reply]
I know this is a stupid followup question, but here goes. Due to the rotation of the fluid, won't there be any "centrifugal force" acting on each fluid particle, and hence, won't the pressure in the radial direction increase? — Preceding unsigned comment added by 117.216.154.47 (talk) 18:34, 30 April 2012 (UTC)[reply]
It's not a stupid question at all. I should have thought of it myself. However, for centrifugal force to result in pressure on the walls of the box, there must be sufficient centrifugal force to overcome internal cohesive (atomic) forces within the fluid, added to the static pressure, forming tiny pockets of vacuum. This could be achieved by making the radial size of the box large enough with respect to the given RPM. If the box is externally constrained from rotating, it may be difficult to get the fluid rotating fast enough. Below the critical speed, there will be no pockets of vacuum and no increase in pressure due to centrifugal force at all. Ratbone124.182.4.180 (talk) 04:17, 1 May 2012 (UTC)[reply]

fever to being cold

If someone went from a fever to being cold constantly and very fast would it be enough to kill (I am asking this out of curiosity). --86.41.70.125 (talk) 14:57, 30 April 2012 (UTC)[reply]

Not sure what you mean:
1) If they go from feeling hot to feeling cold, this can be a sign of disease, but how they feel won't directly effect their health.
2) If the temperature in the environment suddenly changes, this can cause shock, and potentially could kill you do to a heart attack. However, lots of people do just that, like the Polar Bear Club and those who go from a hot sauna to rolling naked in the snow.
3) If you mean your core body temperature suddenly changes by a large amount, that certainly would be unhealthy, perhaps shattering bones and teeth, but this isn't possible. StuRat (talk) 15:39, 30 April 2012 (UTC)[reply]

No I mean if one was to from fever to being cold and then get a fever again and being cold again then it gets repeated over and over again would it be enough to kill them. --86.45.153.97 (talk) 16:13, 1 May 2012 (UTC)[reply]

You just repeated the same thing again. If I didn't understand the first time, then repeating it won't help. Which of the 3 I listed do you mean ? If you mean something else, try explaining it using different words. Specifically, please tell me precisely what is making them hot or cold, and whether they really are hot and cold or merely feel that way. StuRat (talk) 18:26, 1 May 2012 (UTC)[reply]

I mean is someone going from being hot to cold repeatly for a period of time. --86.45.153.65 (talk) 19:57, 1 May 2012 (UTC)[reply]

You still haven't said what is causing them to get hot and then cold. Without knowing that, I can't answer. Is the temperature around them changing rapidly ? StuRat (talk) 20:12, 1 May 2012 (UTC)[reply]

Does the op mean changing illnesses? 99.43.78.36 (talk) 17:44, 2 May 2012 (UTC)[reply]

when the body is under attack there are a number of different defences, you may already know there are white blood cells that seek out foreign bodies and attack them, there are other defences and different levels as well.
Just as humans, plants, animals all die if they are exposed to extreme environments, some of the pathogens (bugs) we get can only survive in narrow temperature ranges. This is not a real problem, because in a 'carrier' (a person who has the illness but does not die from it) the temperature is kept nice and constant and the bug lives a good life, sometimes that person doesn't notice anything. Ideally the bug can pass from person to person and live a good life without those people falling ill, this is common, we have a LOT of 'bugs' in and on us.
when the bug is doing damage and out of control, and the few white cells can't keep up with preventing damage, because bugz are getting mathematically out of hand, then a system wide attack can be conducted. Your body heats up in an attempt to literally cook the bugz, and if that does not work, it will try to freeze the bug to death, and try to repeat that if required. You can die with a fever, or the fever can break and you come back stronger, with an immune system that responds instantly to a known enemy. Hard to say which way things will go, depends on what you have caught. Personally I trust the fundamental systems and if I feel cold I rug and cover up and help my body cook the bad things out, and help freeze on the other hand. If fever is a bad idea paracetamol usually interrupts the natural response and stabilises the temperature, at which point the battle may or may not still be won. you'd be more likely to be a carrier obviously if the bug overwhelmed you and your immune system surrendered after it had a weapon taken away. Penyulap 10:00, 6 May 2012 (UTC)[reply]

Parasitic pepper twin ?

Bell peppers often have something "extra" inside. It looks like a small pepper, itself, and is often tadpole shaped (but curled up). They are usually attached at the stem, but other times are not attached at all, rather rattling around inside. Is this a parasitic twin ? StuRat (talk) 16:11, 30 April 2012 (UTC)[reply]

It's known as an "internal proliferation" and is a form of Parthenocarpy according to this - see also this. Mikenorton (talk) 16:15, 30 April 2012 (UTC)[reply]

OK, thanks for the info. I'll mark this resolved. StuRat (talk) 18:12, 1 May 2012 (UTC)[reply]

Resolved

Engineering at university

At most universities, are engineering modules taught by a wide range of schools/departments e.g. Maths department, environmental sciences department etc or is it all taught by an engineering department? 80.195.94.171 (talk) 16:26, 30 April 2012 (UTC)[reply]

This is very much school-specific. My undergraduate university, which offered accredited degrees in Electrical Engineering and Computer Science, (and many other engineering and non-engineering fields), had certain mathematics courses that could be taken via the math department or via the computer science department. Certain classes were considered "interchangeable," and others required specific departmental course codes. In my graduate program, specific modules were recommended to be taught by individual professors, irrespective of their department. Many professors officially sat in multiple departments, e.g., the physics and computer science and the mathematics department, for reasons of academic politics - so I had the option to list a few courses as either a physics, or as a math, or as a computer science class - even though it was the same material and the same classroom. It's difficult to get statistics for "most" universities, but a good place to begin checking is the accreditation requirements for universities in your region.
For example, my undergraduate institution's accrediting board was the ABET. Here's an overview of what that exactly means: Proud to be an ABET Accredited Program. Roughly, this means that an independent board of academic experts, not affiliated with the university, has certified that our required mathematics and engineering courses are "very rigorous," and meet exacting standards, which-ever department actually is responsible for delivering the instruction. Nimur (talk) 16:55, 30 April 2012 (UTC)[reply]
Here in the UK you'll see an engineering department at most universities, but undergrads can be expected to have to trot to the maths department (and others) for the occasional lecture. LukeSurl t c 23:27, 30 April 2012 (UTC)[reply]
Ex-UK lecturer here: my old department used to provide modules for other departments so we used to have to teach our subject area in other departments. Sometimes this meant I'd have to teach in places such as art studios, machine rooms or draughting rooms. So it's not the case that the students would have to move around: lecturers also moved. I guess it depended on what was easiest to timetable. --TammyMoet (talk) 02:00, 1 May 2012 (UTC)[reply]
When I did my Bachelor of Engineering course, not only were math subjects taught by the Math Department, physics subjects taught by the Physics department, and the one solitary chemistry unit taught by the Chemistry department, in the fisrt two years of the (4 year) course, the university compelled you to do what they called "broadening units" with the stated intent of making you a more balanced man. For broadening units, you could pick whatever units from any uni course you liked, subject to timetable conflicts and subject prerequisites. I chose psychology and journalism units - and never regreted it.
At least here in Australia, universities providing engineering and certain other degrees considered to have academic significance (eg medicine) have for many years compelled undergrads to first do a "common first year", the first year of an Arts degree, or even a whole Arts degree. This is driven because the standard of education in Australian high schools has been slowly dropping for years - I believe the USA also has this problem. High school graduates used to be well equiped to tackle a solid degree, but they are not now.
At the Uni I went to, the allocation of rooms was another matter. I think the central administration used some sort of computer program to allocate rooms, and to the computer a room was a room was a room... We were continually walking from one end of campus to the other. It kept us fit.
Ratbone124.182.4.180 (talk) 03:34, 1 May 2012 (UTC)[reply]
Mechanical engineers often need computer science subjects (for control systems), management is a common topic on engineering courses (for project management etc), and some civil engineers study architecture, as well as the usual mathematics courses. There's a lot of potential subjects, and they may be taught by lecturers from other departments, or on courses run by other departments, or even by engineering faculty/department lecturers. --Colapeninsula (talk) 08:50, 1 May 2012 (UTC)[reply]
In the U.S., at least, I am pretty sure it is similar to the Australian example above. At most universities in the U.S., one would only take the engineering specific courses in the Engineering department. Thus, while a Chemical Engineer would need to take calculus and organic chemistry and fluid dynamics, it would be usual to take the calculus class from the Math department, the organic chemistry from the Chemistry department, and the Fluid Dynamics class from the Chemical Engineering department. This may be more efficient for the university, as Calculus is going to be the same whether it is used by a Chemical Engineer or an Economist, so they can sit in the same room and learn about integrals and differentials next to each other, so there's no real need to teach a sepetate "chemical engineer only" calculus class. --Jayron32 18:14, 1 May 2012 (UTC)[reply]
At my (American) university that I attend now, we do have an engineering section of calculus, with TAs who are engineers, that focuses towards applications of calculus in engineering. At a different school I have attended, they had two sets of chemistry classes, one in the liberal arts school, and another in the engineering/hard science school. In general though, I think Jayron32 is correct, at least in my experience. Buddy431 (talk) 18:40, 1 May 2012 (UTC)[reply]
At my undergrad school, the main required calculus course was taught by the math department and from a "pure" viewpoint. The engineers probably didn't think they needed quite so many proofs, but it was considered important to teach them to them anyway, for culture. --Trovatore (talk) 18:45, 1 May 2012 (UTC)[reply]
It's part of the long and continuous debate in education on the value of a liberal arts education versus a vocational education. The former aims to train a "complete citizen" by giving them the tools they need to do many jobs, and also to be a more complete citizen, fully engaged with the culture, history, and society. The latter focuses purely on skills that train one for their job, ignoring the rest. --Jayron32 23:32, 1 May 2012 (UTC)[reply]
A further comment on something touched on by Nimur above - a reflection on the administrative/political/acreditation quirks of universities - Australian ones at least. I did my bachelor degree in electronic engineering - comprising a common first year (a mix of just about anything, not even engineering of any sort), then 3 futher years specialising in electronics with significant amounts of Math from the Math Dept and some physics from Physics dept. In third year, us electronics students got programmed to do a unit called "Control Systems 304". In Uni nomenclature, the 3 in "304" means a third year unit, the 0 means its supposed to be of only moderate intellectual and workload challenge, and the 4 means something about revison of the unit since the degree was acredited, and whether or not it needs just specific other units done first, or the whole previous year. Electrical students got programmed to do a unit called "Feedback Modelling 316". The "1" in 316 means that the unit is of higher intellectual and workload challenge. Mechanical engineers got to do a unit called something else again. We all went to the same room and got taught the same thing by the same lecturer! Each subject unit was allocated so many points. The points for CS304 were different to FM316. To be awarded a degree, you needed a total of x many points - thus the importance?? of each unit varies depending on the degree you are doing. At exam time, we got to do the same exam papers labelled with all three subject names and codes. So the same subject, actually about various forms of negative feedback, is supposedly easy or difficult depending on what degree your are doing! What utter nonsense. Many other subjects were similar. Keit120.145.52.89 (talk) 04:07, 2 May 2012 (UTC)[reply]

highest decibel rating of any fling insect.

i asked this question in the entertaiment section but i hope i can get also answers here. Wich insect has the loudest decuibel rating when the insect flies? — Preceding unsigned comment added by Saludacymbals (talkcontribs) 20:23, 30 April 2012 (UTC)[reply]

Considering "loudest" to mean the highest decibel rating, I think you are looking for a massive insect that is also in some sense a poor flyer. Echoing some comments from the other Q: As Alansplodge pointed out, a dragonfly can be large, but are amazing fliers and quite quiet. Bumblebees are loud, but are not so loud as a cicada or cicada killer, perhaps in part due to mass differences. I have never heard a Hercules beetle or Goliathus goliatus in flight, but I would think the latter could easily be louder yet, perhaps the loudest. One interesting aspect of your question is that it would be relatively straightforward to gather some big bugs and do some audio testing, but somehow I think nobody has :) SemanticMantis (talk) 00:11, 1 May 2012 (UTC)[reply]
Since none of our photos give a good sense of scale, check out G. goliathus here [2]. SemanticMantis (talk) 00:16, 1 May 2012 (UTC)[reply]

so is even a cicadfa when it flyes loud?(article edited) Secmantimantis i meant any insect! — Preceding unsigned comment added by Saludacymbals (talkcontribs) 10:11, 1 May 2012 (UTC)[reply]

Yes, cicadas are loud flyers. In the USA, it is the loudest insect flight I commonly hear. green fruit beetles are also loud, but I think cicadas are louder. (This is all original research). Among all insects, I guessed the large beetles that I listed above might be loudest. I've looked through the scientific literature, and the only relevant material I found comes from bat research. For example, this paper [3] discusses how insect wing beats reflect sonar signals. This is not the same as loudness, but it's a start. SemanticMantis (talk) 13:12, 1 May 2012 (UTC)[reply]

May 1

Entangled Qubits as Polarized Light

I'm trying to find a way to visualize entangled states, as might be used in a quantum computer. The various articles I can find, and my physics textbook, make vague references to polarized light as a simple model for a qubit. I can understand the one-qubit case, since the components of the electric wave in two orthogonal axes for a coherent beam of light or a single photon will consist of two phasors, represented as two complex numbers, where overall amplitude and phase can be ignored. What's left is the two dimensions of polarization, representing angle and eccentricity. Gates are passing through eg a fluid that rotates polarization, and measurement is passing through a polarizing filter. So a two-qubit register should be two beams of light, or perhaps a beam of light with two colors. I haven't found anything that specifies which it is. Where I get lost is understanding how these bits can become entangled, and where the process can go from there. Black Carrot (talk) 00:30, 1 May 2012 (UTC)[reply]

Size of the cat's eye nebula

Our article says that the core is 0.2 ly. Obviously it doesn't burn like a star, so what defines the core? And how large is the entire nebula? Even a ballpark figure would help me. Thanks, --T H F S W (T · C · E) 05:19, 1 May 2012 (UTC)[reply]

The angular diameter is about 5 arcmin, which corresponds to roughly 5 ly at that distance. The planetary nebula itself corresponds to the bright inner core, so that is material that was ejected at once when the star entered its PN phase. The larger fainter nebula is material that was ejected over a longer period before that, when the star was a red giant. --Wrongfilter (talk) 08:35, 1 May 2012 (UTC)[reply]
So the core is the part we see in all those classic pictures? --T H F S W (T · C · E) 16:16, 1 May 2012 (UTC)[reply]

Air in submerged car

Assuming a normal family car was submerged without it being smashed about, and there was one adult inside, roughly how long could they survive on air trapped in the vehicle?

If they inflated the air bags and punctured them, would that help, and if so, to what extent? --Dweller (talk) 09:36, 1 May 2012 (UTC)[reply]

Airbags don't contain air - they contain the output of gas generators, which are essentially little rocket engines. You wouldn't want to be breathing that. -- Finlay McWalterTalk 09:42, 1 May 2012 (UTC)[reply]
I did wonder! OK, how about the air naturally in the car? --Dweller (talk) 10:37, 1 May 2012 (UTC)[reply]
Since cars are far from airtight the air would remain "trapped" for only a short time. It would probably take only a matter of seconds rather than minutes for the cabin to fill up with water as the air escapes through the door "seals" and other gaps. Roger (talk) 10:46, 1 May 2012 (UTC)[reply]

Interesting. The engine is the heaviest part of the car, so I guess it would nose-dive, leaving whatever air there is against the back window. In a saloon car with no hatchback, there's no door seals there. Would there be as much as a minute's worth of air, or more? --Dweller (talk) 10:49, 1 May 2012 (UTC)[reply]

Chappaquiddick incident#Testimony and cause of death contains some supposition on this subject. -- ToE 11:22, 1 May 2012 (UTC)[reply]

Mythbusters did a number of submerged car escape myths (you can google for videos). Based on their videos the engine pulls the car down nose first and it seems to take about 30-40 seconds for the front to fill and 60-75 total seconds for the air to also escape from the back seat. Of course, the precise time will vary with the make of the car, etc. Dragons flight (talk) 11:50, 1 May 2012 (UTC)[reply]

Have a look at [4] for a practical demonstration. The accompanying notes are interesting. Bazza (talk) 13:37, 1 May 2012 (UTC)[reply]
It's also important to note that you normally do not want to stay as long as possible in the car. Quite in contrary, you should try to open the windows to let water flow inside (so you can open the doors. 188.76.235.0 (talk) 13:49, 1 May 2012 (UTC)[reply]
If car does drop to the point where you can't open the door and your power windows are shorted out and you are in shallow water, then waiting for the car to fill with water so you can open the door may be your only option. Hopefully you could use that time to unbuckle, get into position, calm down and take deep breaths, for your break for the surface once the pressure equalizes. StuRat (talk) 18:10, 1 May 2012 (UTC)[reply]

Googling for something like cubic meter breathe air suffocate suggests that for a person in rest, one cubic meter of air should last about two hours until it has too little oxygen and too much carbon dioxide. Heavy physical activity can cut that time by a factor of ten; I imagine the stress of being in a submerged car will cause you to breathe heavily. If your car's interior volume is, say, three cubic meters, that would be of the order of a couple of hours. If partially filled with water, adjust accordingly. If your head is barely above surface, in a bubble of a couple of bucketfuls of air, let's call it one minute per bucket tops. 88.114.124.228 (talk) 16:13, 1 May 2012 (UTC)[reply]

This formula from an atmospheric sciences course is pretty fascinating. --Sean 20:59, 1 May 2012 (UTC)[reply]
And I think once you're underwater, kicking on the window to break it is much less effective. Ever since reading about it somewhere, I make sure that I keep a sturdy metal instrument in the glovebox with which to shatter the window should something like this ever occur. DRosenbach (Talk | Contribs) 20:15, 4 May 2012 (UTC)[reply]

What does this mean? Gravitational waves should penetrate regions of space that electromagnetic waves cannot.

165.212.189.187 (talk) 13:04, 1 May 2012 (UTC)[reply]

As far as I know, electromagnetic waves, or radiation, can penetrate any region of space. So, the statement does not make sense to me either. Plasmic Physics (talk) 13:16, 1 May 2012 (UTC)[reply]

Well, light can be blocked by solid materials, whereas gravity cannot. That must be what they are referring to. Goodbye Galaxy (talk) 13:35, 1 May 2012 (UTC)[reply]

Electromagnetic waves of frequencies up to up to microwaves are dramatically attenutated by magnetic shielding and by faraday cages - see http://en.wikipedia.org/wiki/Faraday_cage. It is quite simple to make a faraday cage to reduce incident radiation to levels too low to measure. The next thing after microwaves is infrared thru to light - which is blocked by all sorts of things including metals. Gravitational waves are predicted to penetrate all these things. Keit58.164.230.165 (talk) 13:46, 1 May 2012 (UTC)[reply]

Could it be worded better?165.212.189.187 (talk) 14:12, 1 May 2012 (UTC)[reply]

But do faraday cages block ALL radiation?165.212.189.187 (talk) 15:47, 1 May 2012 (UTC)[reply]

Since you can see into a Faraday cage, obviously not. Gandalf61 (talk) 15:52, 1 May 2012 (UTC)[reply]
The blocking of radiation depends on the scale of the mesh. A tighter mesh blocks more ranges of radiation. OsmanRF34 (talk) 15:59, 1 May 2012 (UTC)[reply]
EM radiation is not blocked 100%, it decays at an exponential rate, but never reaches zero completely. Plasmic Physics (talk) 22:08, 1 May 2012 (UTC)[reply]

Touche. I was particularly thinking of a solid one to rule out the light issue. Then the word "should" should be removed bc we know it DOES, right?165.212.189.187 (talk) 17:17, 1 May 2012 (UTC)[reply]

No, we don't know that yet. Gravitational waves have not been successfully detected. Various theories predict that it could penetrate regions impervious to EM waves, but we don't know that for sure yet. 173.32.168.59 (talk) 17:49, 1 May 2012 (UTC)[reply]

Ok I get it. SVT is the best theory so far.165.212.189.187 (talk) 18:06, 1 May 2012 (UTC)[reply]

Faraday cages are not perfect. The scale of the mesh is not the only factor. Since the wires or mesh (or even solid sheet) must in practice have electrical resistance, there will be an excess voltage within the walls and the attenuation imperfect. However, by suitable choice in wire/mesh weight, and double screening if necessary, any required attenuation can be achieved. But the attenuation is not an exponential decay. Exponential decay of EM waves occurs in dielectrics, however, and gravity waves are thought not so attenuated. Ratbone120.145.52.89 (talk) 02:10, 2 May 2012 (UTC)[reply]
Maybe gravity waves can escape from a black hole but light waves cannot? Although I'm not sure about the precise physics of gravity waves (is anybody?). --Colapeninsula (talk) 09:01, 2 May 2012 (UTC)[reply]
The physics of gravitational waves in the classical theory of general relativity is understood very well, although we know this is not the complete story - for that, we would need a theory of quantum gravity. Gravitational waves do not escape from the interior of a black hole, but they are emitted when two black holes (or other massive astronomical objects) orbit one another, or when two black holes collide and merge. BTW, don't confuse gravitational waves with gravity waves. Gandalf61 (talk) 12:02, 2 May 2012 (UTC)[reply]

Amputating limbs of quadriplegics

Do quadriplegics need to have their limbs amputated sometimes? Why doesn't the blood clot, if they necessarily won't be moving their limbs? — Preceding unsigned comment added by 188.76.235.0 (talk) 13:22, 1 May 2012 (UTC)[reply]

Bones make blood. I think they would want to keep limbs where possible. Zzubnik (talk) 14:33, 1 May 2012 (UTC)[reply]
Not sure if that's a valid reason. After all, fewer limbs also mean less blood is needed, so less red-cell generating capacity is OK. Some reasons to keep the limbs might be psychological considerations and the hope that they might one day be cured of their quadriplegic condition, say by use of stem cells to regenerate the nerves. StuRat (talk) 17:59, 1 May 2012 (UTC)[reply]
And they do have problems with blood clots, see Tetraplegia which notes that such people have high rates of Deep vein thrombosis. --Jayron32 16:23, 1 May 2012 (UTC)[reply]

wasp

A few months ago I noticed a wasp in a cupboard in my house. I don't know how long it had been there, but it looked pretty dead to me. I didn't get rid of it because, well, I couldn't be bothered to and it wasn't affecting me. A few weeks later I go in the cupboard again and it's still there, exactly the same as before, it hasn't moved at all. So I am now certain it is dead. Again I just leave it because I'm really lazy. Today, I noticed a wasp buzzing around the house so I opened a window and let it go. Then I thought.. no way, it couldn't be the wasp in the cupboard, could it? I checked and it's gone. So.. do wasps hibernate or something? I'm assuming it was the same wasp, although I can't be certain. It looked like a normal wasp to me and not a big queen wasp. — Preceding unsigned comment added by Waspgirl03 (talkcontribs) 20:32, 1 May 2012 (UTC)[reply]

I doubt if it's the same wasp. It might have been cleaned up by somebody else, blown out of the cabinet when somebody opened the door, or perhaps some other critter dragged it off to eat it. (After it dehydrated, it would be far lighter and more prone to blowing around.) StuRat (talk) 20:43, 1 May 2012 (UTC)[reply]
Nobody else is here but me, and I am certain I didn't accidentally move it. I looked around nearby I didn't find it. Also, it wasn't on it's back it was actually hanging onto a cardboard box in a pose which seemed like it could have been alive if I had seen it outside and moving Waspgirl03 (talk) 21:16, 1 May 2012 (UTC)[reply]
Social insects will routinely "bring out their dead" in order to keep their place tidy. Perhaps you saw the pall bearer leaving. --Sean 21:02, 1 May 2012 (UTC)[reply]
I don't understand. Bring out the dead wasp from my cupboard to somewhere else? For what purpose? And why did it wait several months to do it? Waspgirl03 (talk) 21:16, 1 May 2012 (UTC)[reply]
If they're living up there they would do it as part of general housekeeping. I don't know about the delay. --Sean 14:32, 2 May 2012 (UTC)[reply]
According to various web pages wasps do hibernate, usually in warm sheltered places, and at least in the UK tend to emerge somewhere around mid-April -- so all that seems consistent, but it would have to have been a queen. Looie496 (talk) 21:57, 1 May 2012 (UTC)[reply]
I'm in the UK, so I guess it must have been a queen then. Do the queens look very similar to normal wasps? I assumed they would be a lot bigger Waspgirl03 (talk) 22:01, 1 May 2012 (UTC)[reply]
They're _noticeably_ bigger, but not a _lot_ bigger. This site has a picture of a (very dead) worker and (equally dead) queen next to each other for comparison. See Vespula vulgaris for our article on the beasties, incidentally. Tevildo (talk) 22:21, 1 May 2012 (UTC)[reply]
I figure I should know the answer to this kind of question, since my user name is vespine, (it's not because I'm an expert, I just like wasps, and the word vespine) .. So after reading the wasp article, I think I found the answer: In most species of social wasp the young queens mate in the vicinity of their home nest and do not travel like their male counterparts do. The young queens will then leave the colony to hibernate for the winter once the other worker wasps and founder queen have started to die off. So a YOUNG wasp might not even be fully grown yet, making it probably indistinguishable from any other wasp. Vespine (talk) 02:22, 2 May 2012 (UTC)[reply]
I am inclined to go for 'the same wasp' idea. If it was hanging vertically then this would indicate to a strong degree that it had deliberately placed itself there. Queens hibernate ready to emerge the following spring to start a new colony. Hibernation in many creatures can look very much like the animal or insect is dead by dint of the fact that the body processes slow down and emulate a near death state. The fact that you saw a wasp and the dissappearance of the 'body' seems a moderately high coincidence. Thank you for leaving the sleeping wasp in peace they are useful creatures who have a place in every gardener's heart. Richard Avery (talk) 07:39, 2 May 2012 (UTC)[reply]
Natural selection needs selection for it to work. That wasp will go and breed an army with preferences for house interiors. You should have just killed it. SkyMachine (++) 10:32, 2 May 2012 (UTC)[reply]
Hmm, nice attitude. Since wasps have probably been hibernating in houses since they have existed and they are hibernating for 99.9% of the time they are in the house they would not appear to be a significant threat to the normal house occupier. The idea that this will produce future generations of house-hibernating queen wasps, well, welcome to the present! Richard Avery (talk) 14:57, 2 May 2012 (UTC)[reply]
Well that because people have not been doing their job. We have the power to control these things and select for a world that will not kill us, or even one that will benefit us. How do you think we developed agriculture in the first place. The point is you let the population survive, but you control selection by removing undesirable characteristics present in individuals capable of reproduction. SkyMachine (++) 17:01, 2 May 2012 (UTC)[reply]
Personally I don't consider a single wasp sleeping in my cupboard over winter to be undesirable, I actually kinda like it. I am happy to have given her a home for that time. Anyway, thanks for the answers everyone. Very interesting stuff Waspgirl03 (talk) 22:08, 2 May 2012 (UTC)[reply]
Wow, that's amazingly tolerant. I would kill or eject, if possible, any insect or arachnid or rodent found in my home, as I don't want to share my living space with them. I would leave most of them alone outside, but make an exception for those which can cause injury, like stinging insects and mosquitoes. Those I kill on sight. StuRat (talk) 22:39, 2 May 2012 (UTC)[reply]
If Waspgirl were to be stung or bitten by one of those little devils, it might lower that tolerance level a tad. ←Baseball Bugs What's up, Doc? carrots00:22, 3 May 2012 (UTC)[reply]
Yea, I had one fly down my shirt and sting me a dozen times. This might explain why they must all die, IMHO. StuRat (talk) 00:25, 3 May 2012 (UTC)[reply]
Not all of them, just that one. ←Baseball Bugs What's up, Doc? carrots11:24, 3 May 2012 (UTC)[reply]
There is a certain pleasure in ruining mosquito breeding spots in the spring. Somehow the bites just aren't quite as annoying when you know you've gotten back at them already. Wnt (talk) 18:49, 3 May 2012 (UTC)[reply]
Over here (NZ) there are places named after the abundance of mosquitoes, if you go there for at least two hours, you look like you have the measles, occasionally people develop allergies toward so many mosquito bites requiring medical attention. Plasmic Physics (talk) 21:02, 3 May 2012 (UTC)[reply]

From the 'deep hibernation' behaviour of the specimen which you have described it is very important to be sure. You should carefully check through you whole house, the kind of wasp that you describe can only be one of a few kinds if it is actually the same wasp. I do believe it is a north american native, and if it has made it's way in a shipping container by sea, or otherwise made it's way to the United kingdom it can do very serious ecological damage. The north United States Government centre for disease control has procedures in place for outbreaks of this kind, however if you are in Asia, or even worse, in the United Kingdom, the situation is dire, and I cannot stress this enough, you need to prepare your household according to this guide FIRST. before you inform the authorities. DO IT NOW. Penyulap 10:22, 6 May 2012 (UTC)[reply]

Can't you at least use small font? Nil Einne (talk) 14:56, 6 May 2012 (UTC)[reply]

May 2

Meditation and Sleep

I read somewhere many years ago that someone had done a study on meditation and sleep, and found that if you meditate for half an hour every day, you can get by on four hours of sleep with no ill effects. (My numbers may be wrong. It has been years since I read this, after all.) It seems like everyone has heard something like this, but I've never been able to find the study.

I've taken several Vipassana courses, and Goenka says that anapana meditation can completely replace sleep--but he also says that he can feel his sub-atomic particles vibrating, so I don't consider him a very reliable source. It's the same on Google--many of the people talking about meditation also talk about developing psychic powers and the like, and almost never cite any experiments.

So did any such experiment ever occur? 67.142.166.24 (talk) 00:25, 2 May 2012 (UTC)[reply]

I can see meditation, like any form of rest, slightly reducing the amount of sleep needed at night, but not eliminating it. StuRat (talk) 00:40, 2 May 2012 (UTC)[reply]
It's probably not the study you are thinking of, but http://www.biomedcentral.com/content/pdf/1744-9081-6-47.pdf is a very recent study that addresses the issues and also gives pointers to the earlier literature (not that there is much of it). This is unfortunately an area where many practitioners make wild claims not backed up by solid evidence. Looie496 (talk) 00:41, 2 May 2012 (UTC)[reply]
One of the largest proponents of this must be Transcendental Meditation, some of those loons claim they can fly! Vespine (talk) 01:56, 2 May 2012 (UTC)[reply]
It would take an excedingly well designed and carefully conducted experiment to prove anything anyway, because anyone can train themself to reduce the amount of sleep required. During Word War 2 in the Pacific, soldiers got used to months on end with very little sleep. I enrolled at university when I was working full time - the boss let me take time off during the day to attend lectures providing I came back later in the same day to get my work done. In order to get all the assignments done, I reduced my sleep from 8 hours per night to 6 hours per night without any problems other than feeling sleepy for the first month or so only. I kept this up for 7 years until I had the enginering degrees I wanted. It took about another 3 years until I drifted back to 7 hours per night. I have also found that when changing careers, which I have done 4 times, the amount of time I sleep increases temporarily. So I would think that if you avoid mental stimulation (perhaps by doing too much meditation), you will need less sleep. I know some old retired folks who complain they onnly can sleep 4 hours or so each night. Given that they mostly spend their days sitting around watching television, I'm not surprised. Ratbone120.145.52.89 (talk) 02:23, 2 May 2012 (UTC)[reply]
I'm not sure about any specific studies about getting by on 4 hours sleep, but there have been a fair few studies about Vipassana and sleep e.g. [5] [6] [7]. They show that meditation does affect sleep in some ways, so it's certainly possible that it could reduce the need for sleep, but it seems unlikely you could go without any. From my POV as someone who has also tried Vipassana, I can understand that it might well be feasible to sleep less - if you can't sleep less, then it is hard to see how anyone could maintain meditating 2 hours a day every day. The real problem with meditation though is that we have no real idea what it's doing to our brains (mainly because we don't understand our brains to begin with) so it is very hard to get to the bottom of what effects it has. SmartSE (talk) 10:38, 2 May 2012 (UTC)[reply]
Maybe you could sleep better, if you meditate. Meditation is also a form of relaxation, which will help you fall asleep faster. 88.9.211.140 (talk) 13:18, 2 May 2012 (UTC)[reply]

Four hours per day, or 4 hours per night? In countries were meditation is practiced a lot, it is also usual to take a siesta. Count Iblis (talk) 16:46, 2 May 2012 (UTC)[reply]

I recently read an article — I'm sure in New Scientist magazine (paper edition) but annoyingly I can't now spot it — describing research suggesting that the natural human sleep pattern is better described as two 4-hour sessions than the traditional one 8-hour session, so that waking after 4 hours (or 8) is actually better than after, say, 6 hours. This (if true) would chime with the study the OP recalls. At the time of reading the article I was also struck by its correspondence with the Royal Navy's (and others') traditional Watch system of 4-hour watches. {The poster formerly known as 87.81.230.195} 90.197.66.211 (talk) 19:44, 2 May 2012 (UTC)[reply]
[8] -- Finlay McWalterTalk 22:10, 2 May 2012 (UTC)[reply]

White Blood Cells & infection

Do WBC numbers gradually go down as infections subside? I.e. as the number of infection causing microbes decrease or are there a steady number of WBCs at the infection site until the infection has completely gone, particularly in sterile environments such as urine? — Preceding unsigned comment added by 138.253.210.19 (talk) 12:07, 2 May 2012 (UTC)[reply]

The WBC count can drop rather quickly after antibiotics are started, often within less than 24 hours. Dominus Vobisdu (talk) 15:21, 2 May 2012 (UTC)[reply]

pre-antibiotic era

how did they treat cellulitis in the old days before they had antibiotics? --64.38.226.88 (talk) 12:46, 2 May 2012 (UTC)[reply]

Basically, keep the area clean, washed and properly dressed, apply herbal or chemical remedies, apply cold packs or hot packs, massage or light exercise to stimulate circulation, and hope for the best. This worked best with mild, recently diagnosed cases, of course. Otherwise, if possible, amputation was performed, especially if there was danger of deep tissue involvement or sepsis, which was essentially a kiss of death. The treatment is still basically the same for cellulitis caused by antibiotic resistant bacteria. Dominus Vobisdu (talk) 15:12, 2 May 2012 (UTC)[reply]
Cellulitis is apparently very closely related to erysipela (also frequently caused by Group A streptococcal infection).[9] For this Dioscorides recommended applying the leaves of "POLUGONON ARREN" (identified as "Polygonum-mas [Fuchs], Polygonum latifolium [Bauhin], Polygonum aviculare [Linnaeus] — Knotgrass, Centinode, Knotweed, Armstrong"[10]) Like most of Dioscorides' recommendations, there appears to be at least a grain of truth to this - see PMID 16914113, PMID 19616082, [11] - and like most of his recommendations, no real research seems to have been done in modern times to determine whether it works or not. (On the other hand, apparently Polygonum persicaria can cause erysipelas when used as toilet paper, according to one doctor in 1804. ;) [12]) Wnt (talk) 22:28, 2 May 2012 (UTC) hmmm - didn't search well enough. Also found [13] [14] (which points at panicudine, 6-hydroxy-11-deoxy-13 dehydrohetisane) ; herb a.k.a. Sanguinaria ? (but the link target looks like something else?) [15] Wnt (talk) 22:50, 2 May 2012 (UTC)[reply]
(ec) See all the methods listed under Cellulitis#Treatment, except for antibiotics, of course. Leeches might also have been used, and could help to prevent sepsis. StuRat (talk) 22:32, 2 May 2012 (UTC)[reply]

free space in atoms

if air were free space and a marble was the nucleus of a hydrogen atom and a grain of sand was the electron, would the air inside the atom be the same air indefinitely or would it mix with the air outside the electron cloud in some fashion? — Preceding unsigned comment added by 165.212.189.187 (talk) 12:50, 2 May 2012 (UTC)[reply]

It's fine to scale up an atom to a talkable illustration, but you can't invent a new component and expect to be able to say anything meaningful about it. There is no subatomic equivalent for the air in your illustration (which is 'stuff'); the free space at the subatomic level is 'not stuff', and mixing this 'not stuff' with that 'not stuff' is nonsensical. There's no 'stuff' to mix. — Lomn 13:04, 2 May 2012 (UTC)[reply]

That's what you think. You mean not stuff like anti-matter or not stuff like photons, or like virtual particles?165.212.189.187 (talk) 13:27, 2 May 2012 (UTC)[reply]

If you're thinking of the electron "shells" as being shiny sealed spheres, that is, as Lomn notes, a very false analogy. Those physics textbooks that depict particles as little billiard-ball thingies are, in general, doing you something of a disservice. You're quite right to note that an atom is almost entirely free space - but what can get in there depends on what's trying. Electrons are charged, so when another charged particle encounters the atom, the cross section (the likelihood of interaction) is very high - to a roaming electron, an atom it encounters seems like an unavoidably interacting thing. But to an uncharged particle like a neutron the electrons might as well not exist, and the neutron can sail through without hindrance - only in the fairly unlikely event that the neutron encounters the nucleus will an interaction occur (see Neutron cross section). And to the light, uncharged, weakly interacting neutrino an interaction is even less likely - the average neutrino can whiz through a whole planet (passing untold trillions of electrons on the way) without being deflected from its path at all. -- Finlay McWalterTalk 13:20, 2 May 2012 (UTC)[reply]

No I'm not thinking of shells. if i were I wouldnt have asked the question I would have assumed that it was the same "non-stuff" indefinitely.165.212.189.187 (talk) 13:27, 2 May 2012 (UTC)[reply]

(edit conflict)More than that, there is no inside or outside of an atom, it just thins out like a rain cloud. If you want to be pedantic about it, each atom is the size of the universe. The key is quantum statistics, if you've heard about it - I'm sure someone here would be kind enough to explain it to you. Plasmic Physics (talk) 13:28, 2 May 2012 (UTC)[reply]
Otherwise, I don't understand where you are coming from with the idea of two kinds of free space, and that free space can mix with another free space. Last time I checkedm, free space was an abstract concept. Plasmic Physics (talk) 13:37, 2 May 2012 (UTC)[reply]
The space is occupied by a sea of virtual particles that pop in and out of existance, there is a whole lot happening in empty space. SkyMachine (++) 13:40, 2 May 2012 (UTC)[reply]

I think it is impossible to have meaningful DISTANCE in the universe without free space. Doesn't distance have some correlation with free space? — Preceding unsigned comment added by 165.212.189.187 (talk) 14:42, 2 May 2012 (UTC)[reply]

The oposite is true, for if there was only empty space how could you gauge distance? What would distance mean in an infinite abyss. It is the interaction between different things via forces that creates a sense of scale. The radiation from a star that you detect with your eyes appears to travel at the speed of light. This gives a sense of scale to the space. As does walking across the room using muscle power and friction of the floor, give a scale to the room. SkyMachine (++) 14:51, 2 May 2012 (UTC)[reply]

I didn't mean ONLY free space but the component of it in the mix".165.212.189.187 (talk) 15:07, 2 May 2012 (UTC)[reply]

If things at a distance interact via forces then the interveening space is filled with these force carriers + virtual particles. SkyMachine (++) 15:18, 2 May 2012 (UTC)[reply]
It would be funny if it turned out there really was an Aether. ←Baseball Bugs What's up, Doc? carrots00:19, 3 May 2012 (UTC)[reply]

Doesn't free space enable us to measure the volume of objects? isnt there free space sequestered in objects? wheter it be the same stale space or constantly replacing as a flame?165.212.189.187 (talk) 14:59, 3 May 2012 (UTC)[reply]

No, free space is an abstract concept, like I said. You simply can't handle it in the same way as tangible matter or fields. Plasmic Physics (talk) 20:11, 3 May 2012 (UTC)[reply]
Free space is like emptiness - if you have a chest, which is initially completely empty, and you start adding items to it, the emptiness doesn't go anywhere; the chest is just less empty. Plasmic Physics (talk) 20:18, 3 May 2012 (UTC)[reply]
I remember that not long ago, someone asked whether the universe was expanding by more free space being added. Plasmic Physics (talk) 20:24, 3 May 2012 (UTC)[reply]
I'm not sure about 'free', but the idea that empty space is being added everywhere is reasonable. 71.215.84.127 (talk) 06:19, 4 May 2012 (UTC)[reply]
Everywhere? could you be more vague/specific?165.212.189.187 (talk) 15:35, 4 May 2012 (UTC)[reply]
By what reason? Should I provide a link to the conversation for a better understanding of that discussion? Plasmic Physics (talk) 06:34, 4 May 2012 (UTC)[reply]
Look up Vacuum energy for an explanation of expanding space. Space itself is possibly a computational geometric illusion if the universe is contained in a singularity with no dimensionality. SkyMachine (++) 07:27, 4 May 2012 (UTC)[reply]
How is that relevant to my poorly formed arguement? Plasmic Physics (talk) 08:48, 4 May 2012 (UTC)[reply]
I am not disagreeing with you. The point is empty space does not expand because emptyness is being added, something is added to the vacuum to create this expansion. SkyMachine (++) 09:03, 4 May 2012 (UTC)[reply]
The idea that empty space is (very slowly) being added everywhere is not an inaccurate description of the metric expansion of space. 205.170.21.162 (talk) 22:00, 4 May 2012 (UTC)[reply]

"everywhere" is a strong word. Even in our galaxy, even in our solar system, even inside the sun? Inside my head?68.83.98.40 (talk) 02:53, 6 May 2012 (UTC)[reply]

I don't want to write up a complete explanation on wikipedia I apologise but I would give basic guidance. The marble itself is contains only energy in what you would understand as a pulsating wave, the best visual picture would be the distance that you can stand from a stereo and still hear it, with a reasonably defined boundary. Outside and inside the boundary, of the nucleus is the same substance, space, or air in the example, but inside the boundary there is an energy field, when the core of the energy field is moved about the apparent (but non-existent) surface of the nucleus appears to move. But there is no solid surface of any kind, nor is there any solid interior within the energy field. (the stereo does not exist, only the sound energy field)
A different example would be the solar system, it appears to have form, you conceptualise a disc, even a sphere with a flat central disc within, and at all times people don't mentally draw to scale in their mind, the sun is tiny compared to the whoppers people draw. Imagine our solar system has only the sun and nothing else and you fire an indestructible bullet at it. Hitting anything in particular in the solar system if you fired that bullet would first be incredibly unlikely, further if it did hit the boundary of the sun, it would go into the energy field and zip out the other side like that ship on stargate universe because the energy field called the sun is not solid in any meaningful way. If the solar system were the nucleus, or even if the sun were the nucleus, there is still nothing but space and energy within what is incorrectly perceived as solid matter. Things can either be capable of reacting to particular types of energy fields or not. The comet evaporates, the ship sails through pushing things out of the way, the neutrino sails on through without hitting anything at all, and that is only the energy that is capable of interacting, let alone the energy lacking an order which is capable of reacting. Anyhow, I don't want to get into it in depth, but the marble is just energy like sound on the air, it has a perceived boundary, but no solid form. Penyulap 10:49, 6 May 2012 (UTC)[reply]

Do children of people with speech impediments learn the speech impediment?

And they have to 'unlearn' what their parents taught them in order to speak correctly? --Broadside Perceptor (talk) 16:15, 2 May 2012 (UTC)[reply]

I imagine it depends on the impediment. Stuttering, for example, is not constant, so they also hear the non-stuttered word and can figure out what's it's supposed to be. A lisp, though, might be constant, and so kids might speak that way, too. I suppose once they start spending time with other people they learn the proper way to speak (possibly by being teased when they say things the wrong way). StuRat (talk) 22:56, 2 May 2012 (UTC)[reply]
I certainly think it's possible, but probably rare in this day and age. Most children will be exposed to television and other people, but if the main source of contact with language was with their parents then yes, I don't see why not. Children have an incredible capacity to learn by mimicking, but they don't have any innate sense of WHAT they're supposed to mimic. One thing I found really interesting when I went to Japan a long time ago is how many Japanese people have very heavy American accents when speaking english, even when their english wasn't very good, because the people they learn from and the television shows and movies they watch are predominantly American. Does that count as a speech impediment ;) Vespine (talk) 23:10, 2 May 2012 (UTC)[reply]
Children won't be in contact with just one adult, or to several adults with the same speech impediment, so I don't see the possibility that they develop a speech impediment by learning. I know for sure that children of deaf people are perfectly able to learn to speak a spoken language parallel to a signed language. XPPaul (talk) 23:10, 2 May 2012 (UTC)[reply]
Nell (film) included the premise that a girl was raised in isolation by a single parent with speech problems and she developed her own speech problems and idiosyncratic uses of language that made her difficult to understand. As far as I know the material for this film was pure fiction, but I don't see why it couldn't happen in principle. The necessary isolation would seem to be very rare, though. Most children are exposed to language from many sources. Dragons flight (talk) 00:32, 3 May 2012 (UTC)[reply]
Even if the child had little contact with people other than the parents, which might occur on a remote farm, I doubt the child would end up with a speech problem. It is unlikely both parents would have the same speech impediment. It would be very unusual that an adult with a speech impediment would be unaware of it, and surely, if the child reproduced the impediment, the parent would point out that it is wrong. If both parents have the same attrocious foriegn accent, that could be a problem, but most likely they would speak their own language at home anyway. I had school friends whose immigrant parents refused to speak any language but their own. But my friends grew up without any problem with English - they spoke just like the rest of us. Wickwack120.145.186.64 (talk) 01:17, 3 May 2012 (UTC)[reply]
I had a classmate growing up, both of whose parents who were themselves deaf since birth, and he spoke fine. As noted, people have exposure to language from many sources other than their parents. People's speech patterns are more determined by their peers than their parents, that most people will tend to speak with the accent and dialect of the people they grew up with, and not necessarily that of their parents. That being said, speech impediments can be genetic and hereditary, so it is possible to inherit a stutter or other speech impediment from your parents, but you don't learn such behavior from them, at least not in a lasting way. --Jayron32 01:50, 3 May 2012 (UTC)[reply]
Personal anecdote. I have a bit of a stutter and two kids. I see no evidence of the kids "picking it up" just from hearing me. They imitate me in many ways, including, maybe especially, negative things I sometimes do despite trying not to. But speaking with a stutter is not one of them. And anyway, even if a kid did imitate a parent's stuttering they would quickly find other kids laughing at them and asking why they are doing that. I get laughed at and asked about it by other little kids these days--a strange feeling, I haven't been laughed at by kids for stuttering since I was a kid! Anyway, the reactions of other kids (both malicious teasing and 'good faith' confusion) would make any kid stop it if they are able to. Of course, there are degrees of difficulty--on the milder end it's no big deal and can even be somewhat endearing (as with Bob Newhart). I can imagine a kid, or even an adult, purposefully "cluttering" and mildly stuttering in that kind of way, although I haven't actually heard of anyone doing it on purpose. A couple other points--sometimes stuttering or a tendency toward it can be passed down like other traits. I don't think anyone understands the biological causes of it, but for some people like me there is a physical, neurological aspect (it isn't just psychological), and some evidence that there may be a genetic aspect. Also, lots of kids stutter and then grow out of it--which seems more of a developmental thing. All these things mean it would be quite hard to tell if a kid who stutters, and whose parent does too, is due to developmental, genetic, or imitative reasons. Usually with kids it is a developmental thing. As for other types of speech impediments, I don't know. Pfly (talk) 16:56, 3 May 2012 (UTC)[reply]
yes, but kids usually have many teachers from which to learn, which dilutes the effect. If they live on a desert island and the impediment is difficult for them to master because it requires unique geometry of the airway or vocal cords, and their sloppy imitation of it is accepted by their parents then they will speak with a less pronounced impediment. Penyulap 10:54, 6 May 2012 (UTC)[reply]

attractive force from virtual particles

Force carrier says that virtual particles carry force by transferring momentum from one particle to the other. I can see how that would work for repulsive forces, but what about attractive forces? Bubba73 You talkin' to me? 16:50, 2 May 2012 (UTC)[reply]

Virtual particles are not real, they arise in mathematical computations in an intermediary step and don't refer to physical observables. This means that you now don't have the usual restrictions that apply to physical particles, e.g. they can transfer momentum in a way that would violate causality for real particles. Count Iblis (talk) 17:02, 2 May 2012 (UTC)[reply]
...provided that the final physical observable satisfies the relevant physical constraints (e.g., causality, conservation rules, or whatever, after you sum up the superposition of the virtual particles). Virtual particles don't have to abide by all the same rules as physical particles, but they're not a freebie excuse to violate arbitrary laws of physics in the general case. Nimur (talk) 19:50, 2 May 2012 (UTC)[reply]

bromocresol gren

what is the molar absorption coefficient of bromocresol green --150.203.114.37 (talk) 20:23, 2 May 2012 (UTC)[reply]

This sounds an awful lot like a homework problem. In any case, we have an article on molar absorptivity and the bromocresol green even has a graph of coefficients for visible wavelengths. Per our policy on reliable sources, you should be aware that the graph was made by a volunteer contributor "from experimental data obtained on 1999," (by an otherwise anonymous contributor "User:Nevermore78"), so if that's good enough for your purposes, by all means, have at it! You might consider seeking a more reputable source, though, such as your own lab experiment or a paper published in a source that is generally-regarded as reliable. Nimur (talk) 20:35, 2 May 2012 (UTC)[reply]

Why is it called bromocresol green? After all it is blue when basic and yellow when acidic. Is it green when neutral or somethign --150.203.114.37 (talk) 20:59, 2 May 2012 (UTC)[reply]

It's blue-green when basic. - Nunh-huh 21:08, 2 May 2012 (UTC)[reply]
That doesn't seem like a good reason to call it "bromocresol green". Why wouldn't you call it "bromocresol blue-green"? 150.203.114.37 (talk) 21:12, 2 May 2012 (UTC)[reply]
The question was not what I'd name it, or what you'd name it, but why it is named "bromocresol green". The answer is because it's a shade of green in its basic form. Besides, "bromocresol green" is ever so much snappier than "bromocresol blue-green".- Nunh-huh 21:47, 2 May 2012 (UTC)[reply]
Wouldn't that make it bromocresol aqua? ;) Vespine (talk) 23:03, 2 May 2012 (UTC)[reply]
I guess some scientists are content to tell themselves that it's alright to be short as long as there's some snappiness to make up for it. But we all know that sometimes there's just no substitute for size. "Oh baby, it was so long, I thought I would dye!" DMacks (talk) 00:30, 3 May 2012 (UTC)[reply]
Bromocresol Green is pretty green near it's indicator point - check out this photo. I'm surprised there's no picture in commons of it - I think we have some in the lab, maybe I should take some photos. Buddy431 (talk) 00:40, 3 May 2012 (UTC)[reply]

Lifting buoyant things with just a few drops of water

Let's say I'd want to lift something really heavy (like a shipping container). After making a water tight "fence" around it, with just a few inches between the fence and the (almost empty) container, I fill the room between them with water. Would the container get buoyant because its density is smaller than the density of water? Or would it think: "Archimedes' principle is ok with me, but where's the water I supposedly displaced?" and stay on the ground? If the room between the fence and the container would be extremely small, I'd be able to lift the container by adding a drop of water, and energy preserving etc wouldn't allow that, so that's probably not going to happen. On the other hand, how big should the distance between container and fence be? At what time would the container know it's not a small experiment anymore but more a "container in a lake" situation where it is supposed to be buoyant? I guess it is exactly when there's enough water that can be lifted heigh enough to compensate for the weight of the container, but I'm not sure about it. And what would happen at the border of the inches-situation and the lake-situation? Joepnl (talk) 23:56, 2 May 2012 (UTC)[reply]

No, the drop of hypothetical perfectly fluid water is enough to lift the container, if you manage to make the fence close enough. The energy needed to lift the container comes from the water that runs down underneath the container. Of course, the drop of water has very litte energy, but it also lifts the container only a very very very small distance. More realistically, at very small scales with real water, adhesion and capillary effects will complicate things. But having an inch around the container is plenty to lift it by filling the hypothetical fence with water. This happens e.g. in a lock. --Stephan Schulz (talk) 00:02, 3 May 2012 (UTC)[reply]
(EC)I can't work out why you think this would work but I think you've misunderstood something about boyancy. The container will only ever "float" when it displaces an amount of water greater to its weight, it has nothing to do with the height of the water. After reading the above answer, I don't think the drop will lift the containter at all, even a tiny amount, it won't lift the container until the amount of water put into the container exceeds the weight of the object. The amount of water displaced in a lock is equal to the weight of the object, regardless of how small the lock is. If the lock with the object in it is too small to contain an amount of water which weighs as much as the object, it won't lift the object off the bottom. Vespine (talk) 00:09, 3 May 2012 (UTC)[reply]
No, you are wrong in this. Think about it the other way - start with a bathtub full of water, and a very nearly bathtub-shaped boat slightly smaller than the bathtub, and with a density a little bit less than water. If you put it into the tub, it will sink nearly to the bottom, displacing the water (which runs over the rim and is lost). But the boat will float in the remaining bit of water. And it will do exactly the same if you put the boat into the empty tub and then add the same amount of water you were left with in the first case - i.e. very little water is needed to float the boat. The water that "lifts" the boat does not know if half an inch to the east is the rest of the Atlantic ocean or the rim of the bathtub. --Stephan Schulz (talk) 00:32, 3 May 2012 (UTC)[reply]
No, that is not correct, you need enough water to displace the weight and for the suggested large container, a drop of water certainly will not float it. For instance, you can wade and even sit and lay down in the water at the beach if its shallow enough. But only when the water is deep enough can you float. The displaced water in the fenced area or the lock must be of sufficient volume to force the initial drop underneath it, since this larger body of water acts as a counter-weight to the ship's weight. --Modocc (talk) 00:50, 3 May 2012 (UTC)[reply]
I think the confusion here may arise from how we are thinking about the seals in the system. For instance, Fig. 6 shows a common illustration of an arrangement where hydrostatic pressure can be used to create a sort of "leverage" effect. So you can use a mouse to lift an elephant, if the seals are all perfect. If there is a perfect seal, then I think a drop of ideal fluid could lift arbitrarily large masses by a small amount, provided that it is supplied from below with enough pressure. This may not be what the OP meant, but I think it might help reconcile the different answers above. SemanticMantis (talk) 01:17, 3 May 2012 (UTC)[reply]
In reply to the bathtub example above, maybe what you aren't considering is that the water will spill OUT of the bath tub as you drop the object in, by the time most of the object is in the bath, most of the water will be OUT of the bath and the weight of the object will now greatly exceed the weight of the water left and it will indeed sink all the way to the bottom. Now if you had a bath tub with walls extending upwards so the water wouldn't spill over the sides of the bath, the water will instead spill over the sides of the bject and it will still sink. Vespine (talk) 01:24, 3 May 2012 (UTC)[reply]
Stephan Schultz is correct. Vespine and Modocc didn't understand the mention of locks. The arbitarily small amount of water will work just fine in lifting the object. The mass of water used has nothing to do with it. Seals have nothing to do with it - seals are not required. Think of it this way: Consider a bathtub three-quarters full of water. Place in it a shoebox sized object with a density (say) 90% of water. It will obviously float, with approx 10% of its volume above the water surface. In floating, it displaces 90% of its volume, so the water level in the bath rises slightly - some fraction of a millimeter. Now, construct around the floating shoebox sized object a thin rigid wall just slightly larger than the floating object, being carefull not to touch the object, so that you end up with the shoebox sized object floating in water contained within a thin "boat" floating in more water which is in the bath. Now, place a support post, that just fits, under the thin "boat". Now you can drain the water out of the bath. You still have the shoebox sized object floating in water within the "boat" - it has not moved up or down. And the water within the boat has a volume much less than the displacement volume! It works because atmospheric pressure on the water allows the mass of the floating object to be transfered to the container. Ratbone58.164.234.216 (talk) 01:37, 3 May 2012 (UTC)[reply]
Oh snap! Good explanation Ratbone. Now to work out wtf I was thinking. The 90% as dense as water thing maybe did it for me because a 90% water density object will NEVER sink, even if you DRAG it underwater; I was thinking of sinking boats the whole time, for some reason. maybe the "heavy shipping container" thing threw me, but of course the container still must be less dense then water or else it won't float to begin with! I was obviously a bit denser then that in this case ;) Vespine (talk) 02:02, 3 May 2012 (UTC)[reply]
Ok, I was definitely wrong, I'm not denying it, but I think I worked out my confusion; because there MUST be a point where the object you are trying to float WILL squeeze the water out of the "bath" and NOT float, the unstated premise is that the container it self must be at least up to the waterline of the object! Right? Which ratbone's great example illustrated clearly. If the container isn't up to the waterline of the object, then the object WILL touch the bottom, which is a bit of a tautology really. 02:16, 3 May 2012 (UTC)
There is a limit, which is controlled by the cohesion of the water molecules to each other, and the number of molecules in a droplet. If you spread your floating layer of water molecules to single-molecule thickness, it will seperate to the point where it will no longer support your object. As long as you have enough water to maintain cohesion, an object will float on it. But there is a minimum. It is much smaller than intuition would indicate, but it is there. --Jayron32 05:28, 3 May 2012 (UTC)[reply]
Physically, capillary force will limit it. If the water layer is thin enough, the object will be pulled very strongly to the bottom, squeezing out water. The force gets stronger as the gap closes; if the surfaces are accurately the same shape essentially all the water can be forced out, and intermolecular forces between the two surfaces take over, producing a strong bond between the two.--Srleffler (talk) 17:02, 3 May 2012 (UTC)[reply]
Is that true? I would have expected it to pull water into the crack between the object and the bottom... Wnt (talk) 20:37, 3 May 2012 (UTC)[reply]
(ec)A drop of water is not sufficient to float a large ship, nor will a dozen water molecules float a thimble, but I was wrong too, for the largest ships' displacements in these locks are larger than the remaining volume. The ships do need clearance, thus in addition to adequate depth, there has to be an adequate water volume to separate the objects. --Modocc (talk) 06:17, 3 May 2012 (UTC)[reply]
Ratbone, something is wrong. Consider your system, before you add the "boat". You write "In floating, [the object] displaces 90% of its volume, so the water level in the bath rises slightly - some fraction of a millimeter." This glosses over an important question: how much does the level in the bath rise? Since the object displaces a fixed volume of water (by the law of buoyancy), the change in the level of the bath depends on the size of the part of the tub that is not occupied by the object. If the tub is smaller, the displaced volume of water must rise higher.
When you add the thin "boat", if nothing moves the height of the water inside of the boat depends on the size of the original tub. This is clearly a contradiction, so something has to move—either the object changes height within the boat or the boat changes height within the tub.--Srleffler (talk) 17:02, 3 May 2012 (UTC)[reply]
The fact that you don't need the displaced water can be verified by doing this experiment: Float a wide base plastic cup in a slightly larger bowl and compare the weight of the cup with that of the water you used. --Modocc (talk) 18:08, 3 May 2012 (UTC)[reply]
The problem you are describing is called Archimedes paradox. Our article agrees with the first response above, by Stephan Schulz. --Heron (talk) 18:46, 3 May 2012 (UTC)[reply]
Cool,the problem is "official" :). Still not completely convinced though. See the picture.
Archimedes
Archimedes
. The grey thing is a very heavy container (but its density is lower than that of water). At the bottom, the container balances on a very small (1 cm thick, perfectly square) stick (the scale of this work of art isn't correct, imagine a very small line). Now I add a liter of water. As we already found out, the container doesn't mind if it floats on an inch of water or an ocean, and makes room for the water by going up. The room at the sides is only a micrometer wide so the only place for most of the water to be stored is at the very bottom. A liter contains 1000 cm3 water, so the container would be lifted 10 meters (33 ft). So by lifting 1 liter of water let's say 20 meters and pooring it into the structure, I'd be lifting a huge weight 10 meters up. I guess I just made an error, but if not: after this I attach a rope to the structure, connect it to a generator, release the water and the container will go down again. The generator would generate much more electricity than I needed to lift a liter of water. Joepnl (talk) 23:05, 3 May 2012 (UTC)[reply]
The energy in and out should be about the same (minus loss in your output), for you are lifting a small amount of mass, water, a long distance and dropping a larger amount of mass, the container, a short distance. The energies are the product of the masses and the distances. --Modocc (talk) 23:24, 3 May 2012 (UTC)[reply]
That's the reason for this Reductio ad absurdum. I only lift the water, say, 20 meters. But in turn, the water lifts the container 10 meters. That's 1 kg x 20 m and say 50,000 kg x 10 m. It should be equal, which is what I don't understand. Joepnl (talk) 23:39, 3 May 2012 (UTC)[reply]
Side note: if I used a pump to put in the water at the very bottom, I probably would have to a use a very strong pump, supposedly having the same end result as just pooring some water on top of the structure. Joepnl (talk) 23:56, 3 May 2012 (UTC)[reply]
Either your 50,000 kg container is too dense so it is not buoyant or you are not using enough water to fill the separation underneath. Moreover, the separation is the entire area underneath the container and not just the middle extension as you might be thinking. --Modocc (talk) 00:18, 4 May 2012 (UTC)[reply]
Modocc now has it right. The error made by Joepnl is that while he/she contrived a long thin stick in a bottom cylinder so as to produce a large rise while not needing any much volume of water in the cylinder, he forgot to estimate the volume created, as the large heavy floating object rises, directly underneath that floating object - the volume of water thus accumulated co-axial with the long thin extension. Also, Joepnl is one of a number of posters who are focused on whether a small volume of water can lift a large volume object, floating in a just large enough rigid container, though a large distance. Well, it can't, but that is NOT part of the original posters's question. The OP only asked if you could get it to float. Floating is floating regardless of whether it rose off the bottom 1 meter, 100 m, or 1 mm. NOTE: Even though the water required to lift the object a significant distance is not insignificant, it can still be a heck of a lot less than the volume of the object. Ratbone58.169.234.93 (talk) 01:29, 4 May 2012 (UTC)[reply]
At this stage, unless you are comfortable with the Archimedes Paradox article, don't post unless you have tried an experiment with real objects. A plastic cup - say the typical sort with sloping sides - within another cup will do fine. Put some water in the inner cup with water as well to increase its density. Ratbone58.169.234.93 (talk) 01:29, 4 May 2012 (UTC)[reply]
Joepnl was the original poster, so he was asking a followup question, and I have agreed with you whole heartily as to his mistakes, but we shouldn't be discouraging postings unless they are disruptive. --Modocc (talk) 01:50, 4 May 2012 (UTC)[reply]
Erk! Yes, so he was. I shall spank myself. Ratbone58.169.234.93 (talk) 02:33, 4 May 2012 (UTC)[reply]
I guess I should be silent for quite some time now.. Please consider the last question as never have been asked. Thanks everyone for the explanations! Joepnl (talk) 01:50, 5 May 2012 (UTC)[reply]
There is a simple and general solution to the apparent "paradox", which only assumes constant density. If the heavy object rises then there is a space A near the top of the container that was originally occupied by water and is now occupied by the object. And there is a space B near the bottom of the container that was originally occupied by the object and is now occuiped by water. Spaces A and B have the same volume - call this V. And suppose the centres of gravity of spaces A and B are separated by a vertical height h. Then the gain in potential energy of the object is ogh where ρo is the density of the object. And the loss in potential energy of the water is wgh where ρw is the density of water. So the net energy gain is V(ρow)gh. But because the object floats we know that ρo < ρw, and so this net energy gain is negative i.e. energy is lost when water is added to the container and the object rises up to a new floating position. Another way to see this is to realise that pushing the object back down again to its original position will require doing work against the buoyancy of the object, so (potential energy of new floating position) - (potential energy of old "pushed down" position) = -(work done pushing object down) < 0. Gandalf61 (talk) 08:21, 4 May 2012 (UTC)[reply]
Nice, Gandalf61. My inattentive thoughts were initially too impetuous, but the Archimedes Paradox has been much easier to think about that way when the objects are fully submerged like that, such as explaining the separation of various volumes of oil and vinegar. --Modocc (talk) 11:52, 4 May 2012 (UTC)[reply]

May 3

Who would and wouldn't want to live forever

Can psychologists ever say, with as much certainty as the experimental sciences ever give, whether or not a person would want to live forever given the choice (given that they aren't already attempting suicide)? If so, does this suggest a limit on the computational diversity of human minds (e.g. via the obvious argument involving mind uploading and the undecidability of the halting problem)? NeonMerlin 08:05, 3 May 2012 (UTC)[reply]

That needs some clarification. Would everybody be living forever? My dad outlived most of his peers, and declared that life became pretty boring because all his old friends were dead. HiLo48 (talk) 08:33, 3 May 2012 (UTC)[reply]
I don't think they would have studied it much given at this point that everyone dies or is expected to die. If you give people the option of unlimited lifespan at peak mental and physical health than the answer is going to be different to giving the option to extend life of a person in ever declining health. SkyMachine (++) 08:51, 3 May 2012 (UTC)[reply]
I would like to live for a long time, but not forever. If you lived "forever", you would spend most of your time alone, floating in complete darkness because all the suns would have gone out and all atoms would have decayed. Zzubnik (talk) 10:02, 3 May 2012 (UTC)[reply]
You could always get around to try & replace the existing universe with a new one if it ends its usefulness. If a digital copy of a mind decided it did not want to continue then that mind can be replaced by an archived copy of the mind in a state that wanted to continue on. All the archive copies would have to be destroyed if a true and final end is sought. SkyMachine (++) 10:07, 3 May 2012 (UTC)[reply]
It seems to me that how you handle memories would play a role here. If we retain the current limit on amount of memories, this means we would either need to stop memorizing new things or forget old things. Either one would be like having a form of dementia. StuRat (talk) 15:45, 3 May 2012 (UTC)[reply]
Say what? We forget old things all the time. We may not forget them completely, but details fade over time and are replaced with vague impressions that presumably take up less space. --140.180.5.49 (talk) 17:00, 3 May 2012 (UTC)[reply]
Right, but now say you live 1000 times longer. You'd then only remember 1/1000th as much of each year, on average. That's so little, most years would be lost entirely. StuRat (talk) 02:11, 4 May 2012 (UTC)[reply]

I regard myself as an immortalist. I want to live as long as possible. Maybe not literally forever, but as close as I can get. Abyssal (talk) 16:31, 3 May 2012 (UTC)[reply]

I bet what you're really saying is "I want to live as long as I can with a decent quality of life". Would you wish to have Stephen Hawking as a role model, for example? He has spent many of his 70 years with a form of MND. There is a Greek myth about a woman who asked for eternal life but forgot to ask for eternal youth. --TammyMoet (talk) 20:15, 3 May 2012 (UTC)[reply]
Tithonus, incidentally. Tevildo (talk) 21:31, 3 May 2012 (UTC)[reply]
See also the first paragraph of Cumaean Sibyl#Later literature for another classical example. Deor (talk) 22:44, 3 May 2012 (UTC)[reply]

Programmed death

Everyone dies so one can speak of a biological program for decay to death of the body's physical and mental functions that starts inevitably from the instant of birth. Is the rate of dying fastest while one is awake and active physically and mentally, or while one is sleeping?

This morbid question is prompted by the observation that some cells are programmed to die during early development of the embryo to open the spaces between fingers and toes, and a story by Edgar Allen Poe about a dying man kept alive by hypnosis. 84.209.89.214 (talk) 11:46, 3 May 2012 (UTC)[reply]

The only "biological program" that controls death and decay that I know of is telomere decay, which affects how many times your cells can duplicate and replace themselves, which puts a finite limit on your life as an organism. My guess is that being awake or asleep doesn't affect this too much on average; ditto active or sluggish. But I don't really know. There are a lot of other factors regarding whether one lives a long time or not beyond telomere decay — you only have to worry about your telomeres if nothing else gets you first! For most people, death comes a lot sooner than that. --Mr.98 (talk) 14:02, 3 May 2012 (UTC)[reply]
A bit off topic, but is there some study that has determined what percentage of people die simply of old age (i.e. not mayhem, accidents or disease)? Clarityfiend (talk) 23:45, 3 May 2012 (UTC)[reply]
Exercise and thinking involves burning blood sugar, which tends to cause oxidation aging damage to the cells. Anti-oxidants can help reduce this effect, but not eliminate it. A near starvation diet also limits the amount of oxidation (and exercise and thinking) your body can do. StuRat (talk) 14:59, 3 May 2012 (UTC)[reply]
This is largely the same debate as planned obsolescence, I think, except that the body of course was not "designed" to work or "planned" to fail in the quite the same sense. Wnt (talk) 15:39, 3 May 2012 (UTC)[reply]
But the result is the same, if evolution favors species with programmed death (since that allows for shorter generations and thus the ability to more quickly adapt to a changing environment). StuRat (talk) 15:42, 3 May 2012 (UTC)[reply]
There is also a cost/benefit tradeoff involved. Mechanisms that lead to eventual cell death may in the shorter term improve odds of survival (or perhaps reproduction). The net effect may be improved survival of the genome. I have in mind the fact that telomere decay is a primary defense of the body against cancer. Uncontrolled cell division due to genetic damage is a big problem. Telomeres are kind of a "dead man switch" that shuts down the process if other controls fail.--Srleffler (talk) 17:10, 3 May 2012 (UTC)[reply]
Damage will accumulate in proportion to the rate of metabolism. Thus the damage rate ought to be lower while sleeping when metabolism is lowered. Plus for humans we sleep indoors for the most part and at night typically, thus exposure to UV radiation and environmental pollutants ought to be lower while sleeping. SkyMachine (++) 21:04, 3 May 2012 (UTC)[reply]

People are bringing up oxidative damage, but no study has ever demonstrated that the normal rate of it actually shortens your lifespan. Clearly the resulting mitochondrial mutations and increased rate of protein turnover probably aren't good for you, but that doesn't rule out that something else ruins your cells long before that extra damage matters. The best evidence for that theory is the extended lifespans of animals on super low calorie diets, but it seems plausible that's just a natural starvation response, and not an absence of damage. Antioxidants can certainly benefit various medical conditions, but there's no proof this is by lowering the rate of oxidative damage. In fact, there's no evidence that dietary antioxidants significantly reduce oxidative stress in the human body. Someguy1221 (talk) 01:39, 4 May 2012 (UTC)[reply]

Very laid back Bluebottles, how to distinguish true C. vomitoria?

Hi. This may seem like an odd question, but it's a genuine mystery for me. Whenever I've encountered flies of most descriptions during my life, they seem very edgy, like you get within 30cm of one and it takes off, not wishing to get squished. For 3 days however, I've been encountering some exceptionally laid back, almost what I'd describe as "stoned" blue bottle flies (Calliphora vomitoria). If they were anymore laid back, they'd be practically horizontal. You can approach them, pick them up and move them, put them out of a window, etc, and they make no attempt whatsoever to fly off, they don't buzz, they don't crawl away even. They just sit there. And this puzzles me. Is there any way of distinguishing if these are true C. vomitoria, or if they are a mimic species? A fly that doesn't fly away...it should be called a "walk" :^/ (Edit: One other thing, they also appear "pumped up", like they've been taking steroids or something.) MarkBurberry32|talk 19:58, 3 May 2012 (UTC)[reply]

Whenever I've come across slow insects it's because they're not warmed up enough to react quickly. Insects are cold-blooded and need a certain amount of ambient heat to function properly. I don't know where in the world you are, but certainly in the UK recently it's been cool enough to affect insects. --TammyMoet (talk) 20:11, 3 May 2012 (UTC)[reply]
These have been inside a house, TammyMoet, where the heating has been on during the day for the last few days - set at around 20°C/68°F. I can't think of any reason why they would be cold. MarkBurberry32|talk 20:23, 3 May 2012 (UTC)[reply]
I had a problem some years ago with repeated infestations of bluebottles. As it turned out they were laying eggs in a cavity wall and getting in via gaps in my Skirting board. They were very inactive which I was told was because they had recently 'matured' after pupating. If I was away from home for a few days they would be fully annoying bluebottles that couldn't be swatted without the reflexes of a ninja. Can any entomological experts confirm that young flies are lazy? 87.115.34.230 (talk) 22:39, 3 May 2012 (UTC)[reply]
Your explanation seems a very good candidate once temperature is ruled out. The other possibility is an old fly, perhaps of last autumn's generation. Blue bottle fly describes that they can hibernate as pupae or adults. SemanticMantis (talk) 23:18, 3 May 2012 (UTC)[reply]
I think they are normal blue bottles that are slow for some reason. Why would any other species mimic a housefly? You could easily confirm that it is a fly at least by checking the mouthparts and halteres, though rigorous species-level ID is quite challenging. WP:OR: I've seen similarly slow blue bottle flies and common houseflies in the fall, but never a large pulse like you describe. SemanticMantis (talk) 23:25, 3 May 2012 (UTC)[reply]
Right, so these are likely to be recently pupated bluebottles, which are in essence, slow, because they're flat out pooped from pupating or maturing? Cool :) It just seemed really odd to me to find a fly without the will to fly, so to speak. Thanks for the information guys, I appreciate the help. MarkBurberry32|talk 23:31, 3 May 2012 (UTC)[reply]
If you haven't seen them pupate, check it out: [16]. Seems very stressful indeed. The head blows up like a balloon to generate the necessary pressures! SemanticMantis (talk) 00:03, 4 May 2012 (UTC)[reply]
Bloody hell! That looks about as exhausting as you could possibly get. No wonder the poor buggers are flat out tired! Kind of reminds me of myself, as a positively fat bloke, trying to get out of a polar sleeping bag at 5am on a camping trip. Not a pretty sight :) MarkBurberry32|talk 00:51, 4 May 2012 (UTC)[reply]
Too bad the flies didn't think to add a zipper, like in your sleeping bag. StuRat (talk) 02:06, 4 May 2012 (UTC) [reply]

May 4

All-axial or all-equatorial?

Do scyllo-inositol and β-hexachlorocyclohexane have a structure with each substituent on the cyclohexane ring taking an axial position or one with each substituent taking an equatorial position? Whoop whoop pull up Bitching Betty | Averted crashes 00:56, 4 May 2012 (UTC)[reply]

In general, equatorial is more stable than axial, so scyllo-inositol (like the structurally similar glucopyranose) will likely be all-equatorial. While there might be specific interactions with the chloro groups which might change the behavior from glucose, given the size of the chlorines and the close interactions that the all-axial configuration would impose upon them, my inclination would be that β-hexachlorocyclohexane would also take an all-equatorial conformation. -- 140.142.20.101 (talk) 01:22, 4 May 2012 (UTC)[reply]

Correct. β-hexachlorocyclohexane (CHCl)6 is all equatorial doi:10.1039/P29760000614, as is scyllo-inositol (CHOH)6 doi:10.1021/cg060179a. --Ben (talk) 08:46, 4 May 2012 (UTC)[reply]

How fast it takes for Venus to lose its atmopshere

Venus don't have a magnetic field/magnetosphere, I wonder how Venus maintains a thick atmosphere of 90xs the pressure of earth. I heard Venus was once a blue planet, it end up to have a runaway greenhouse effect which adds 100xs the atmosphere from 3 to 4 billion years ago. If sun brightens by next few billion years, will Venus' atmosphere erode slowly or quickly no later than 2 billion years. In other words will Venus lose most/all its atmosphere quickly by just 1 to 2 billion years, or Venus and Earth will erode its atmosphere about the same time, it will take time (slowly) for Venus to lose all its atmosphere.--69.226.42.182 (talk) 01:51, 4 May 2012 (UTC)[reply]

The standard model for this is called gas escape, and the formula is essentially solving for the thermodynamic equilibrium of a Maxwellian gas subject to gravity. You can find more rigorous treatments, including models that deal with solar wind, ionization, magnetospheric trapping, and all sorts of other parameters. Our article lists several advanced references. I also refer to de Pater and Lissauer as my usual first-place-to-check for these sorts of questions. I think planetary atmospheres are Chapter 3, there's a looooong discussion of atmospheric evolution over cosmological timescales. Nimur (talk) 03:05, 4 May 2012 (UTC)[reply]
Sorry, Chapter 4, especially 4.8, though there's discussion of atmospheres in radiative equilibrium in Chapter 3, and extensive coverage elsewhere. Nimur (talk) 03:09, 4 May 2012 (UTC)[reply]


Isn't Venus's atmosphere being replenished by volcanism? Venus has periodic episodes of planet wide flood basalts, similar to but much larger than the flood basalt that created the Siberian Traps. I've read somewhere that Earth will get an atmosphere like that of Venus today due to increasing solar activity in the distant future. Count Iblis (talk) 02:45, 5 May 2012 (UTC)[reply]

Toxicity of lead anti-radiation materials

Why, despite its toxicity to humans, lead is still being used for radiation protection? Is there a non-toxic material, which is as effective as lead or even more?-- 176.241.247.17 (talk) 13:44, 4 May 2012 (UTC)[reply]

Lead is only toxic if you ingest it (eat, drink, breathe). In the cases where its used to absorb radiation (lead aprons for patients receiving x-rays, lead plates protecting radiologists) the lead is properly encapsulated and poses no danger at all. Depleted uranium is considerably more effective at absorbing X and gamma radiations, is again only toxic if you somehow get some of it inside your body, and is used in similar circumstances to lead, again properly contained, without issue. -- Finlay McWalterTalk 13:49, 4 May 2012 (UTC)[reply]
Lead is extremely dense which is useful for making a good shield, and at about 1 US$ / pound it is still relatively cheap. Comparable shielding made out of more common materials, like steel or concrete, would be 50% heavier and 3 to 6 times thicker. Lead is very useful when you want the shielding that is not too heavy or too bulky. As Finlay says, lead is not particularly dangerous unless you ingest or inhale it, which can generally be prevented by encapsulating the lead in other materials. Dragons flight (talk) 14:08, 4 May 2012 (UTC)[reply]
Lead also stops most of the nasty stuff - alpha particles, beta radiation, gamma ray emission, high energy ionizing electromagnetic radiation, and so on. Other materials stop specific types of hazardous radiation: paraffin can stop slow neutrons, and even ordinary paper will block alpha particles, but lead blocks almost everything. That makes it great for places where you have technicians who don't know the physics behind the radiation source; or when you don't have the specialized equipment necessary to detect and classify the type of potentially-hazardous invisible radiation. In certain specialized environments, like labs, reactors, and so forth, different shielding is used that is appropriate to the specific circumstance. For example, my school's nuclear reactor used water: as both coolant and shielding. For the ~ 1 megawatt PULSTAR, water is both safe and effective: but in general, unless you have an experienced team of nuclear engineers and physicists, plus the necessary equipment, to validate it, you can't be sure if water is effective shielding from your radiant energy source. Lead, as a "lowest common denominator," is the general failsafe for most cases. Nimur (talk) 14:56, 4 May 2012 (UTC)[reply]
(ec)To put down a few numbers: Density of lead is 11.34 g/cc, for steel is 7.75 to 8.05 g/cc. But according to File:Ironattenuation.PNG iron has a photon mass attenuation coefficient around 0.04 for many high frequencies. I found a university poster about lead shielding which drops nearly as low at a moderate frequency [17] but it looks like there are some lower values where it has an advantage. Should find better data but it looks like the difference might be more complicated (more directed toward some particular range of energies) than I thought... (Oh, that mass attenuation coefficient article shows File:Photon Mass Attenuation Coefficients.png, which is interesting - makes me wonder why they don't use an alloy... or would that not actually work in terms of covering the gaps?) Wnt (talk) 15:04, 4 May 2012 (UTC)[reply]
anti? pro? lead rant and soapbox
The following discussion has been closed. Please do not modify it.
I'm always much amused by the concern taken by individuals, organistaions, and governments over lead.
Not because I don't think it is toxic (that seems well established), but because it is so ubiquitous in the environment. You need to keep things in perspective. Lead used as shielding in nuclear facilities is not going to get into people & hurt them. Lead used as shielding in medical facilities is not going to hurt anybody, as it is inside plastic or metal covering. Nuclear facilities are not that common anyway.
Decades ago, upon testing for lead in children, they found it, at significant levels. They decided that paint was the problem, and in my country, lead has been banned from paint for decades. Still, whenever somebody sampled for lead in children, they still found it at presumably significant levels. Of course, if some young couple buy a cheap old house, and decide to sand it all back to bare timber for a nice DYI paint job, they spread that 60 to 90 year old lead around again... Then experts decided that lead in gasoline was the problem, so they banned lead in gasoline, 20 years ago. Still, whenever somebody sampled for lead in children, they found it, you guesed it, at much the same levels. I worked for >40 years for the phone company, mostly in areas concerned with cables. From the 1920's to the 1970's they used lead sheathed cable for all underground trunk and distribution cabling. The sheath when new was about 2.5 mm thick. The cables were pressurised with air by cylinders or compressors in each exchange. The air consumption was minute. Almost all of these cables were still in use in the 1990's and even later. The air consumption had become very high, requiring compressors running a lot of the time. Why? The old lead sheaths had become very thin, paper thin in some cases, so thin they are porous to air. You tell me - where has all that lead gone? Any every time some developer was to convert an old industrial or commercial estate into residential, and someone decides to take soil samples, they find lead .... 121.221.216.254 (talk) 16:07, 4 May 2012 (UTC)[reply]
Cute rant, but lead levels in children have declined steeply with the removal of lead from gasoline. Putting lead into something that was burned by every vehicle in the country was, well, a documentably bad idea. --Mr.98 (talk) 16:31, 4 May 2012 (UTC)[reply]
And a short sharp reply to my rant. Trouble is, there are statistics, and there are those that mis-use statistics. Your graph plots two things: 1) the number tested over the last 13 years, shown as increasing. Provided the statistical sample is large enough (which would seem to be the case), the number tested is irrelevant. What is relevent is that in my country and many others, lead was removed from gasoline over 20 years ago, not 13. Actually, in my country, it was removed 24 years ago. 2) the second and most important thing plotted in your graph is the number of children found over an (unspecified) level. This means nothing. If I plot a graph of people with more than a certain level of viruses in their bodies, by setting the level I could produce a result (number of people) varying from near zero to 100%. What you SHOULD graph is the % blood level by mass, or % tisue level by mass, of lead averaged over all children tested. You might find that the figure hasn't changed much. Children are not unaffected if the level is just below an arbitary criteria, they aren't suddenly significantly affected if they are just over an arbitary criteria. 120.145.32.74 (talk) 01:40, 5 May 2012 (UTC)[reply]
I agree, that CDC graph is useless. It is a plot of children found over a totally arbitary value of 10ug/litre. It could have resulted from, for instance, a blood level averaged over all children dropping from 10.0 to 9.9 ug/litre. It is well known that lead in gasoline was just one of a multitude of sources. The Europeans, in wanting to bring lead levels in blood down, have tried all they can. 10 years ago they banned the use of lead in solder - surely a minute use compared to industry generally, and how on earth does the lead in solder, generally well inside electronic devices, get into children? They went on to ban the use of lead totally, though they had to allow the continued manufacture of lead acid car batteries - there just isn't a reliable, low cost, safe, and effective alternative. Outside of Europe, lead is present in all manner of things - including lipstick and other sorts of makeup. Wickwack124.178.63.42 (talk) 02:57, 5 May 2012 (UTC)[reply]
I can't speak for Europe, but in America the statistics are extremely clear: since Lead paint was banned in 1978, along with the gradual phasing-out of Tetraethyllead as a gasoline additive from the late 1970s onward, there has been an extreme drop in blood levels of lead in young children (ages 1-5). To assert that there has been no reduction is ludicrous. -RunningOnBrains(talk) 03:31, 5 May 2012 (UTC)[reply]
Here is another article summarizing some studies not covered in the linked graph. Every single study I can find (and there is are a lot of studies on the first page of simple google searches) shows a sharp drop in blood lead levels in every age group studied. I have no idea where you got the idea that blood lead levels have not been decreasing, but the data shows you are incorrect. -RunningOnBrains(talk) 03:42, 5 May 2012 (UTC)[reply]
I will note that legislation barring or restricting the use of lead-based solder doesn't always have to be based in sometimes-questionable think-of-the-children reasoning. Eliminating lead-based solder may also reduce occupational exposures by individuals who work with it, reduce the amount of lead-containing waste that ends up in landfills, and cut back on lead exposure for the third-world workers who recycle our first-world electronics waste. TenOfAllTrades(talk) 03:50, 5 May 2012 (UTC)[reply]
The articles cited by Runningonbrains are good, because they show data on actual blood levels (ug/L) (as well as the count above the arbitary 10 ug/dL limit). They do indeed show a roughly 5-fold reduction over 30 years in the USA. My (Australian) view that eliminating lead from gasoline had little practical effect stems from reading articles in Australian newspapers that lead me to believe the lead levels in blood in children are as bad as ever - in fact it has been a major controversy in Western Australia in recent years, in part because birds have been dying due to lead poisoning. However I have been unable to find any thing I can provide a link to, other than than limitted example surveys of counts over the silly 10ug/dL limit. Wickwack124.178.63.42 (talk) 04:01, 5 May 2012 (UTC)[reply]
A lovely distraction, but utterly and obviously a soapbox. When in doubt of statistics or facts, just rant some more. This is both off-topic at this point and the ranter in question has yet to cite a single fact for his silliness and has admitted that they have no references for their ridiculous assertions. I'm hatting this. --Mr.98 (talk) 13:18, 5 May 2012 (UTC)[reply]

Boiling water

(In case you're not familiar with the matzo ball recipe here's one.) The matzo ball mix is formed into somewhat dense balls of the ensuing goop and dropped into a pot of boiling water, at which time the balls enlarge and become fluffy after about 7 minutes. What is it about boiling water that makes it work? Scientifically speaking, why doesn't this happen in still cold water? And if the matzo balls float (which they do), how does the 30-40% of the ball that sits above the surface cook? I'm assuming it's because it soaks up the water like chromatography. Is it the heat that causes the dough to expand and fluff? Sorry for asking like 5 questions in one. DRosenbach (Talk | Contribs) 19:34, 4 May 2012 (UTC)[reply]

It's the baking powder that causes the expansion. The sodium bicarbonate in baking powder decomposes in an acid-base reaction which releases carbon dioxide gas to form bubbles in the batter. Heat accellerates the decomposition of sodium bicarbonate. Roger (talk) 19:45, 4 May 2012 (UTC)[reply]
But they fluff even without the baking powder (although not as much) and what you're saying seems to suggest that the balls would fluff even if just placed in cold water, yet that doesn't happen. But thanks for the heat-related info. DRosenbach (Talk | Contribs) 20:21, 4 May 2012 (UTC)[reply]
Cold water would most probably dissolve the balls back to a (lumpy) batter if left long enough. Heat is needed to accelerate the bicarb decomposition - cold dough doesn't form bubbles at a high enough rate. Recipes that don't use baking powder must have another leavening agent instead. Roger (talk) 20:54, 4 May 2012 (UTC)[reply]
As Roger explains, it's not the water, it's the heat. Boiling water is extremely efficient at distributing heat evenly around the matzo ball, and keeping it at an almost constant 100 °C (212 °F), which keeps it warm enough to accelerate the reaction but cool enough so the dough doesn't burn. I'm sure there are other substances that you could use instead, maybe some sort of oil that boils around the same temperature, but I doubt you'll find one as unbiquitous, affordable, and non-toxic as water. As for fluffing without baking powder, I'm sure there are some other basic substances in the recipe which cause the same reaction, but at a much lower yield. -RunningOnBrains(talk) 22:21, 4 May 2012 (UTC)[reply]
See also: dumplings. As with any cake or bread, air pockets will expand the mixture as it's cooked, because warm air expands, and because water escaping as steam does the same. Extra gas from baking powder or yeast simply add to this effect. 86.140.54.3 (talk) 16:22, 5 May 2012 (UTC)[reply]
I think you intended to link to Matzah ball? Nil Einne (talk) 16:44, 5 May 2012 (UTC)[reply]

Hollywood etymology

What is the etymology of Hollywood ? --Pixeltoo (talk) 20:16, 4 May 2012 (UTC)[reply]

Holly + wood. The name was coined by the real estate developer H.J. Whitley, presumably in reference to the California holly. LANTZYTALK 20:43, 4 May 2012 (UTC)[reply]
This article claims it comes from a pun on something a Chinese laborer said to Whitney, but that seems a little contrived to me, personally. This should really be on the Humanities or Linguistic desks; science it ain't. --Mr.98 (talk) 20:46, 4 May 2012 (UTC)[reply]
The name was originally applied to one particular real-estate development, but later applied to that suburb of Los Angeles. From there, since movie studios sprang up in that suburb, it came to mean "American films". Ironically, the studios have now largely moved out of Hollywood, to other LA suburbs, but the term remains intact, even copied, in terms like Bollywood and Dollywood. StuRat (talk) 22:21, 4 May 2012 (UTC)[reply]
Actually, per Hollywood#History, the name "Hollywood" was applied to the area long before the development which commissioned the Hollywood sign (which was actually named "Hollywoodland" (picture). The name is used repeatedly in this 1903 publication by the Historical Society of Southern California. The name supposedly came from Ivar Weid, a prominent investor in the area in the late 1800s, but since that part of the story is cited to an offline book, I have no way of verifying. -RunningOnBrains(talk) 22:35, 4 May 2012 (UTC)[reply]

2 questions about a chemical solution

1.if the window stays open, a wasp or a fly could enter to the new room. there are types of flies that are very fast and are not easy to be killed swiftly. also a Anisoptera entered, and i had to kill her, sadly. it's such a beautiful animal. anytime i used Nerve agent to paralyze them from the very-fast flying speed.

what i ask is a method of getting the animals out to the open without (!!!) killing them (i would pass that if i can), and it also brings me to the next question in the subject:

2.how did people in ancient times fought such "intruders"? let's say i am an ancient Egyptian farmer living in the Nile delta, and anytime i get 4-6 of them into my dwelling. or let's say i am something else.. Aboriginal, or Thai, or whatever, in Ancient times. what methods and chemistries used by our ancestors to do this job?.

thanks. — Preceding unsigned comment added by 79.176.176.145 (talk) 20:59, 4 May 2012 (UTC)[reply]

I think the easiest way to get insects out of a building without killing them would be to use a net to catch them. As to what ancient people did to insects, I guess the same as most modern people: just hit them with an object to kill them. Or if there were many of them, maybe a net may have been more effective. I don't see why they would bother to use chemicals. - Lindert (talk) 21:47, 4 May 2012 (UTC)[reply]
Using nets inside seems risky. I picture lamps being smashed. StuRat (talk) 22:13, 4 May 2012 (UTC)[reply]
In the US we use window screens to keep insects out, and are amazed that many other nations don't. They keep out larger insects, although some get in when you open the door, and small insects, like fruit flies, can go through the screens.
For slow-moving insects, like many beetles, I can capture them by placing a glass over them on a wall, then sliding an index card or playing card between them and the glass.
For faster insects, a possibility might be to cool them down first, using a CO2 fire extinguisher, which will slow them down enough to use my method with the glass and card. (It won't require very much CO2, but they will recover quickly.)
Note that I don't just use the catch and release method for the insect's well-being, it also keeps disgusting bug guts off my wall. Capturing them with a glass and card is less work than cleaning the wall (especially considering that I will then have to clean the entire wall, since the "clean spot" will otherwise stand out. StuRat (talk) 22:03, 4 May 2012 (UTC)[reply]
I once again suggest my mother's favorite solution to this problem: a can of spray starch. Takes 1 second to dry, the insect falls to the floor unable to fly, and can be safely ejected in a manner of your own choosing. The spray is non-toxic, doesn't stain, and any overspray is easily wiped up with a damp rag.
--DaHorsesMouth (talk) 23:11, 4 May 2012 (UTC)[reply]
Insect repellent has a section on natural repellents. You could try spreading some of them around the window. As for in the past First Nations and Inuit had ways of keeping bugs away. See Insect Repellents and Insecticides, though bear fat may be in short supply, and here. Given that I would assume that the people in other countries had traditional methods for keeping pests at bay before modern times. CambridgeBayWeather (talk) 06:46, 5 May 2012 (UTC)[reply]
I think they just became accustomed to insects there. Heck, even in farms with livestock, everybody gets used to the flies. And before insects were known to spread disease (especially mosquitoes), they were just considered an annoyance. StuRat (talk) 06:54, 5 May 2012 (UTC)[reply]
I found these just now, All Natural Insect Repellent through the Ages: A History of Bug Repellent, A brief history of insect repellents, part 1, Earliest Known Bug-Repellant Plant Bedding Found at South African Rock Shelter, History of Insect Repellents and [www.researchinformation.co.uk/pest/2001/B106296B.PDF Insect Repellents – Past, Present And Future]. While I'm not sure that some of those would be classified as reliable sources the last one probably would. CambridgeBayWeather (talk) 15:14, 5 May 2012 (UTC)[reply]

Lions not on the Gulf of Guinea?

According to File:Lion distribution.png, Panthera leo was not historically present on most parts of the Atlantic coast of Africa between the mouths of the Congo and Mano Rivers. Why not? Nyttend (talk) 23:04, 4 May 2012 (UTC)[reply]

Lions live in the savanna, while those areas are tropical rainforest. The map File:800px-tropical wet forests.png has a green area showing the tropical rainforests in Africa. Notice how well it matches your map, but in reverse. Lions don't live in the forests. --Jayron32 23:54, 4 May 2012 (UTC)[reply]
Except for one very rare species. Clarityfiend (talk) 08:40, 5 May 2012 (UTC)[reply]
I was quite unaware that the southern DR Congo was predominately not rainforest. I was under the impression that most of the pre-European country (as well as northern Angola) was heavily forested, rather than savanna. Therefore, although I knew about the rainforest on the Gulf of Guinea, I didn't make the connection. Nyttend (talk) 12:43, 5 May 2012 (UTC)[reply]
What was the dominant large predator in that jungle region? SkyMachine (++) 20:25, 5 May 2012 (UTC)[reply]
Probably the African leopard. Unlike lions, leopards are much more adaptable in their habitat preference. While lions are really restricted to savanna, leopards will pretty much range anywhere, from desert to rainforest and everywhere in between. --Jayron32 20:36, 5 May 2012 (UTC)[reply]

May 5

Dimensions of inflation

What was the diameter of the universe just before and after Inflation (cosmology)? 71.215.84.127 (talk) 01:09, 5 May 2012 (UTC)[reply]

See Shape of the Universe. The short answer is that we don't have a good idea about the diameter of the universe at any time. It may not have a meaningful diameter. --Jayron32 01:28, 5 May 2012 (UTC)[reply]
From Inflationary epoch: "This rapid expansion increased the linear dimensions of the early universe by a factor of at least 1026 (and possibly a much larger factor), and so increased its volume by a factor of at least 1078."
"[T]he visible universe ... radius after initial inflation was (very roughly) 10 cm...compared with 14 billion light years now."[18] (emphasis added)
If those are both correct then the initial radius was about 10-17 Angstroms, right? 71.215.84.127 (talk) 05:37, 5 May 2012 (UTC)[reply]
Well, that depends on whether you're asking about the whole universe, or just the visible universe. Although even in the case of the visible universe, I'd be hesitant to simply apply the scale factor. Given how non-intuitive general relativity is on some issues, I'm not certain that the visible universe is still quite the same thing before and after expansion. That is to say, particles that are in your visible universe prior to inflation may be outside of your visible universe following inflation. Maybe someone better at physics can enlighten me. Someguy1221 (talk) 05:59, 5 May 2012 (UTC)[reply]
1026 is a lower bound, not an estimate. It's the smallest expansion factor that's consistent with the observed flatness of the universe. It doesn't come from any theoretical model of the actual inflationary process; those tend to give much larger expansion factors. I doubt the size of the visible universe before inflation is a meaningful quantity; in any case we don't know what it is. There may not have been a beginning of inflation. -- BenRG (talk) 21:50, 5 May 2012 (UTC)[reply]
There was a question elsewhere about "empty space" or some such. Supposing one could somehow travel at nearly limitless speed and could somehow get to a point where all the galaxies are behind you. In front of you would be solid black, right? With no apparent end to it? ←Baseball Bugs What's up, Doc? carrots03:44, 5 May 2012 (UTC)[reply]
That depends. Do you think that the universe is ekpyrotic? If so, then yes, unless our big bang is not unique (see cosmological principle.) Otherwise it depends on whether the topology is closed, as Jayron suggested; if so then no and if not then yes. 71.215.84.127 (talk) 05:29, 5 May 2012 (UTC)[reply]

Also note that the universe is about as flat as a pancake under some models (flatter, in fact). Collect (talk) 12:29, 5 May 2012 (UTC)[reply]

Can you link to an article about such models? I can't imagine how that could work. Nyttend (talk) 12:44, 5 May 2012 (UTC)[reply]
There's some information in the article (already linked above) called Shape of the Universe. --Jayron32 19:34, 5 May 2012 (UTC)[reply]
That's flat in the sense of flat (Euclidean) three-dimensional space, not flat in the sense of having only two dimensions. -- BenRG (talk) 21:50, 5 May 2012 (UTC)[reply]
Where the internal angles of a triangle several billion light years across sum to 180° - SkyMachine (++) 23:22, 5 May 2012 (UTC)[reply]
The universe is and always will be infinite by our understanding of the term. Small minds create absurdly flawed theories and models which, with no exception, rely on shit at some critical point which they often call something like a black box or unknown, it's like when you give an idiot a tangled ball of fishingline, they can't possibly untangle it, and so just try to tidy up, gathering out long 'useful' bits, and crushing the rest into as small a ball as possible. Limited universe theories always without exception offend reason at some point when you follow the long threads they do show you back further, the closer you get to the beginning the more obvious they have no idea what to make of the central mess. Anyhow, there is no idiot with a tangled ball of fishing line that won't shout he knows all about how to untangle it, just ask them, they're experts. Penyulap 11:07, 6 May 2012 (UTC)[reply]
this sounds just like someone shouting that he just untangled the largest ball of fishing line in the boat.68.83.98.40 (talk) 12:49, 6 May 2012 (UTC)[reply]

Article about rain

Hi - There's been some discussion on the Talk page about "Rain" concerning my opinion that the article does not *actually* discuss why rain occurs. It gives the *appearance* of doing so, but does not, actually. This is a little surprising to me. I wonder if somebody could look into this? I think it could greatly improve the article if there were a good, easy-to-understand yet thorough, explanation of why rain occurs in the first place. (Something involving thresholds, gravity, temperatures, air pressures, "dew points," the size of the drops, the composition of clouds and why and how they "hold" the water, etc.)

Again, if you read the article carefully, you may agree that it *appears* to give an explanation of why rain forms and falls, but does not actually do so. — Preceding unsigned comment added by 68.196.248.241 (talk) 05:03, 5 May 2012 (UTC)[reply]

I have had a quick look at Rain. It already has Good Article status. Most Wikipedia articles, even the best ones, can be improved by addition of a little extra information here, or an improvement in the syntax there. Often, the first step in the improvement process is when one User clearly and precisely identifies a problem. The second step is when another User, or even the first one, then makes suitable to edits to eliminate or mitigate the problem. The allegation here is a most general and imprecise one - that the article appears to be scientific, but is not actually scientific. Criticisms of this kind are unhelpful and most unlikely to bring about any improvement. If the User at 68.196.248.241 is willing to use the Talk page to clearly and precisely identify a problem there is a possibility another User might take up the opportunity to fix the problem. If the User at 68.196.248.241 is not willing to assist in this way then it is highly unlikely he will ever see an improvement unless he does it himself. Dolphin (t) 05:25, 5 May 2012 (UTC)[reply]
I'd post this on the talk page for Rain. StuRat (talk) 05:27, 5 May 2012 (UTC)[reply]

DVD player

Why can't scientists invent a DVD player where you just put the disc in and it plays - don't have to wait for all of that opening stuff and don't have to select play from the menu. Just bypass the menu - put it in and go. Also, how about one that would be as easy to fast forward and rewind as a DVR? Bubba73 You talkin' to me? 05:14, 5 May 2012 (UTC)[reply]

For the first part, many Disney DVDs do that. They figure they want kids to pop it in and have it play. I've never had any trouble with FF and REW. You hit the buttons and it starts to fast forward or rewind slowly, and push it again and again to speed up, then play to resume normal play. What's hard about that ? StuRat (talk) 05:24, 5 May 2012 (UTC)[reply]
It takes a long time compared to my DVR. Also, the DVR has a button that you can press and it goes back a few seconds. A DVR is a lot smoother operating that way than any DVD player I've had. Bubba73 You talkin' to me? 05:32, 5 May 2012 (UTC)[reply]
Well, it does take some time to spin the DVD up to the proper speed, but only a second or two in my experience. Ultimately, I expect the portable video format to be something more like a flash drive, which should be quicker and more flexible. StuRat (talk) 05:37, 5 May 2012 (UTC)[reply]
Also, because a DVR is reading from a hard drive with high rotational speed (7200 to 10000 rpm, it can quickly jump back to any given point in its memory, where as DVDs (as with other heavier physical media) are subject to higher rotational latency, since they typically only spin between 600 and 1500 rpm. Although now that I'm looking at the figures I'm not sure it would be a noticeable difference. -RunningOnBrains(talk) 06:05, 5 May 2012 (UTC)[reply]
The licencing terms imposed by the DVD Forum do not allow consumer equipment or software to bypass opening sequences. Please report to your nearest media consumption re-education booth for suggesting that this is not entirely suitable. 71.215.84.127 (talk) 05:34, 5 May 2012 (UTC)[reply]
DVDs which go right to a menu don't much bother me, either, as I often want to change spoken language or subtitle settings. What I do hate is DVDs that feel the need to play a video clip after each click on the menu. That gets annoying quickly. StuRat (talk) 05:29, 5 May 2012 (UTC)[reply]
The comment two above mine is the key to your first question — they can develop it and have developed it, but they're not allowed to implement it for most DVDs. Nyttend (talk) 12:46, 5 May 2012 (UTC)[reply]
So what happens if they do it anyway ? How about a DVD player manufacturer in a nation with no enforcement of such laws, like China ? StuRat (talk) 02:09, 6 May 2012 (UTC)[reply]
If they do it anyway, potentially they'd be sued for violating the terms of their licence or at least have the licence revoked (meaning they can't legally produce DVD players anymore). As for China, it's likely there's some enforcement in China, however it's possible such DVD players do exist. It doesn't help people in other countries though, unless they're going to try and important the DVD player themselves. Nil Einne (talk) 04:40, 6 May 2012 (UTC)[reply]
An extraneous "ant" seems to have crawled into you last sentence. :-) StuRat (talk) 06:18, 6 May 2012 (UTC) [reply]

Is Planetary Resources making a space VLBI telescope array?

This blog post suggests that Planetary Resources is working on space VLBI telescope array based on Émile Henry Labeyrie's "hypertelescope" design. Is there any corroboration for that? 71.215.84.127 (talk) 05:47, 5 May 2012 (UTC)[reply]

I can't find any.
That said, the test array in LEO is very easy to arrange though. The L2 array is fringe in a way, one gust of solar wind can blow it all away. There are craft heading for the moon that can give a lift to hitch-hikers like these, and there are pushes to send people there which are spoken of, but second priority to Mars, so it's harder to predict that it will happen, I'd give it a 10% in 20 years chance for L2 and a 80% chance for LEO in 15 years. The chances that anyone will make billions is Zero. The chances that anyone will make a similar space based array for the same purpose is 100%. Penyulap 20:33, 5 May 2012 (UTC)[reply]
Do you think the "interstellar missions.... allowing the capability of exploring other planetary surfaces" of http://lmr.nasa.gov will image the surfaces of extrasolar planets with lunar VLBI first? 71.215.84.127 (talk) 22:21, 5 May 2012 (UTC)[reply]

Over-active Immune System

Is a person with an over-active immune system, i.e., multiple allergies, less likely to become sick from common illnesses such as colds and flu?

Also are they less likely to contract cancer? Thanks, Wanderer57 (talk) 17:00, 5 May 2012 (UTC)[reply]

I would think they might be more resistant to infectious disease, although if they have inflammation and breaks in the skin from scratching, this might make it easier for microbes to get in. As for cancer, that seems to work by fooling the immune system into thinking cancer cells are good cells, so having a stronger immune response isn't necessarily going to improve the ability to recognize cancer cells. StuRat (talk) 17:08, 5 May 2012 (UTC)[reply]
Yes and no. Cause and effect may not have a bearing, but the environment that causes one will cause the other. For example, multiple allergies are common in highly developed countries where people eat highly processed foods and are exposed to a wide variety of chemicals, the same places that people have the highest possible cancer rates. Avoiding the risk factors for one will avoid the risk of the other, so leading an Amish or permaculture lifestyle would decrease both, and living in the city would increase the risk of both.
The immune system learns from what it has seen before, the people who were old enough to survive Spanish flu never contracted swine flu, there was a simple cut-off date for elderly patients in the hospitals that doctors noticed, but as a general law of the universe, however sick you are, you can always get a little more sick, it's called Murphy's law and believe me, I know it's true, the only advantage to laying in a hospital bed is that you are slightly less likely to be hit by cars, that's pretty much it, everything else is a downside. Penyulap 21:13, 5 May 2012 (UTC)[reply]
There seems to be a statistical link between allergy and protection against cancer: A Danish study found a link with breast and non-melanoma skin cancer and contact allergies; people suffering from allergies had a lower risk of developing those cancers. http://healthland.time.com/2011/07/12/the-curious-link-between-allergies-and-cancer/ Ssscienccce (talk) 08:08, 6 May 2012 (UTC)[reply]
When it comes to the common cold, there's no advantage at all. We all get colds, but whether we notice or not depends on the reaction of our immune system. Ssscienccce (talk) 08:13, 6 May 2012 (UTC)[reply]

Pronunciation of vitamin?

Any ideas when and why the different pronunciations of the word vitamin emerged in American and British English? I think it would be good to explain this briefly in the article, supposing good references can be found. --John (talk) 18:58, 5 May 2012 (UTC)[reply]

You might have more luck pn the Lamguage Desk.--Shantavira|feed me 19:25, 5 May 2012 (UTC)[reply]
Thank you, I have linked to this discussion from there. --John (talk) 19:38, 5 May 2012 (UTC)[reply]

This is a bit OR, but the OED lists the "vittamin" pronunciation first. However, it then says, "orig. vitamine", with the pronunciations in the opposite order. Perhaps then the UK pron came later, as the word became more established and thus more anglicized? (On the other hand, the UK pron is what one would expect for a Latinate word; it's the US pron that's irregular.) — kwami (talk) 20:23, 5 May 2012 (UTC)[reply]

Why is "the UK pron . . . what one would expect for a Latinate word", Kwami? My dictionaries indicate that vitamin was formed from Latin vita and International Scientific amine, and the i in Latin vita is long; so in my understanding the traditional English pronunciation of the Latin word would be /ˈvaɪtə/—cf. vital, etc. Perhaps the pronunciation of victuals (from a related Latin word with a short i) as "vittles" had some influence. Deor (talk) 23:37, 5 May 2012 (UTC)[reply]

Temporary change in sexual orientation

(This was inspired by a question on the humanities desk.) I believe that as of now, there's no way to permanently change a person's sexual orientation. However, suppose I'm gay and wanted to be straight. Would there be any drugs I could take to decrease my attraction to males, and increase my attraction to females? I know there are drugs that can decrease/increase libido, but how about selectively changing one's libido?

Note that this is purely out of curiosity. I'm not gay, and even if I were, I wouldn't inject random chemicals into myself. --140.180.5.49 (talk) 19:01, 5 May 2012 (UTC)[reply]

No. there is no such drugs. SexoPharmachology is only about ADULT Physiological arousal --- at it's Sexual context. this arousal is achieved Via Sexuali-attracting Stimuli (which could even be a "Sex-doll" if u have a Philia for it), and what is considered Sexual Stimuli is DICTATED (!), by the very stubborn NeuroPsychological mechanism of "Sexual orientation". therefor, changes, even temporal, are unlikely to occur. Blessings. Babaluba100 (talk) 19:13, 5 May 2012 (UTC)[reply]
See gay bomb. The reverse has been tried by the military. It didn't work, of course. -RunningOnBrains(talk) 19:33, 5 May 2012 (UTC)[reply]

Hormone replacement drugs used by people who have sex changes alter many aspects of their brain and thinking. The stereotypes that are factually based for men and women's cognition, such as women are better at having 5 conversations at one time, and men are better at reverse parking, actually reverse themselves as those drugs take effect. A man who is taking women's hormones actually loses his ability to reverse park and can concentrate on cooking whilst chatting on the telephone (according to studies I've seen). Sexual drive is effected by drugs and hormones, steroids, menopause and so on, so messing with hormones is going to have some effect, how pronounced will vary from patient to patient.

Permanent changes would be the result of permanent changes to the hormones in a persons body. Castrati are people who have had permanent changes to their bodies and hormone levels, with corresponding changes to libido, and injuries to critical systems like the pituitary gland will wreak havoc as well. Penyulap 21:00, 5 May 2012 (UTC)[reply]

Once an adult, hormones can no longer change your sexual orientation. They could only have an affect while the brain is developing. Thus, from before birth to a bit after puberty, with earlier exposures more effective. StuRat (talk) 23:01, 5 May 2012 (UTC)[reply]
in addition to sturat: the in-development affect of hormones is more on gender identity, and seems to influence on Sexual orientation more INDIRECTLY, than directly..
The flaw in the OP's premise is the notion that it's either-or, like an on-off switch. Lots of folks are on a sliding scale, and can adopt a "straight" lifestyle if they want to. That's not a "cure", it's just a lifestyle decision. Obviously, someone at or near an extreme end of the sliding scale is not likely to be very successful at trying to be something they're basically not. ←Baseball Bugs What's up, Doc? carrots00:12, 6 May 2012 (UTC)[reply]
Anyone can adopt a straight lifestyle, but that doesn't mean their attraction to the opposite sex suddenly goes up, or that their attraction to the same sex goes down. I'm asking about actual changes in attraction, not just forced changes in behavior. --140.180.5.49 (talk) 00:49, 6 May 2012 (UTC)[reply]
Changes over time are certainly possible. Are you attracted to exactly the same type of person now as you were 10 or 20 years ago? ←Baseball Bugs What's up, Doc? carrots18:23, 6 May 2012 (UTC)[reply]

Neutrons

Since neutrons can pentrate most things including quite a few feet of concrete, how is it possible to make a neutron reflector?--92.25.96.129 (talk) 21:10, 5 May 2012 (UTC)[reply]

Your text got a little bit garbled there, How you make it would depend upon it's purpose, if it is for a science experiment on a lab desk or as part of a building, or part of an environmental measurement device. Penyulap 21:18, 5 May 2012 (UTC)[reply]
(corrected initial post) I have no intention of making anything, I just wondered how neutrons can be refelected when they pass thro most things?--92.25.96.129 (talk) 21:29, 5 May 2012 (UTC)[reply]
Different substances have different scattering cross sections with neutrons of various energies. When that cross section is large, elastic scattering is more likely to occur. 71.215.84.127 (talk) 22:26, 5 May 2012 (UTC)[reply]

pearsons r and F TEST

When computing an F test and Person's r, if significance testing is conducted for both tests using the same set of data, why is it possible to achieve contrary results? why we are getting different results — Preceding unsigned comment added by 206.54.215.76 (talk) 23:03, 5 May 2012 (UTC)[reply]

The F test is sensitive to the number of degrees of freedom. Describe your data and model in more detail to get a better answer. 71.215.84.127 (talk) 23:10, 5 May 2012 (UTC)[reply]
Can you explain exactly what it is you are doing? Pearson's R isn't a statistical test, it's a measure of correlation... --Tango (talk) 23:21, 5 May 2012 (UTC)[reply]
Isn't r squared equal to variance accounted for? An F test is the ratio of explained variance to unexplained variance, or the ratio of between-group to within-group variance, per the F test article. Edison (talk) 02:33, 6 May 2012 (UTC)[reply]

May 6

Chemical smells

Do amines smell different, depending on whether they are primary, secondary, or tertiary? How do they compare with imines? Plasmic Physics (talk) 01:00, 6 May 2012 (UTC)[reply]

I'm asking because, butan-1-ol, ethoxyethane, butanone, and butanal all smell different from each other. Plasmic Physics (talk) 02:18, 6 May 2012 (UTC)[reply]

I would imagine that each smells distinct, though it may take some training to tell them apart. --Jayron32 02:28, 6 May 2012 (UTC)[reply]
Why would it be so much harder? Plasmic Physics (talk) 11:12, 6 May 2012 (UTC)[reply]
I didn't say it would be any harder than anything else. Just that, exactly like anything else, you'd need some learning in the matter, such as reading the label, smelling it a few times, and getting used to the differences. Pretty much exactly like learning any smell. --Jayron32 12:33, 6 May 2012 (UTC)[reply]
Smells depend ultimately on olfactory receptors. Humans have close to 1000 distinct olfactory receptors, each with its own idiosyncratic sensitivity to some aspect of molecular structure. So it is extremely difficult to predict in a principled way which types of smells will be distinguishable from which other types of smells. Looie496 (talk) 17:46, 6 May 2012 (UTC)[reply]

Where does the vitamin D in polar regions come from?

The inuit get their vitamin D from eating fish and seals. The fish and seals get their vitamin D from their diet, and ultimately all vitamin D found in nature is produced from UVB radiation. But in the polar regions, the Sun is so low in the sky that all the UVB radiation is filtered out of the sunlight before it reaches the round. So, where does the vitamin D come from? Count Iblis (talk) 01:21, 6 May 2012 (UTC)[reply]

You need very little UVB to make vitamin D, and luckily, they get very little UVB. StuRat (talk) 01:53, 6 May 2012 (UTC)[reply]
The text at Vitamin_D#Production_in_the_skin directly contradicts Count Iblis's assertion that not enough UVB exists in polar regions. I have no idea who is right and who isn't, except to note that the Wikipedia article (and presumably the references) says that it isn't a problem. --Jayron32 01:59, 6 May 2012 (UTC)[reply]
We had a question related to this a while back, and I worked out at that time that based on the literature, a sun angle of about 30 degrees above the horizon is required for significant vitamin D production. If that's right, then in the far north there will be part of the year where there is no production, but only part of the year. Looie496 (talk) 03:39, 6 May 2012 (UTC)[reply]
Our article at the current time seems to support this view. CI's claim is unclear, it seems to suggest year round there is no production whereas Jayron32 may have simply been pointing out our article suggests there is sufficient opportunity to for vitamin D production in spring, summer and autumn. (Our article specifically notes that during the summer some northern parts of Canada have greater levels of UVB then in the equator.) Nil Einne (talk) 05:33, 6 May 2012 (UTC)[reply]
I believe you find quite a bit of vitamin D naturally in fish, which is incidentally a common source of food for natively northern cultures. Someguy1221 (talk) 04:21, 6 May 2012 (UTC)[reply]
I think the question here is how it occurs 'naturally'. If CI's claim is true and all vitamin D is produced by UVB (which our article supports) the question still remains how the fish get it in the first place, somehow it must come from UVB. But considering the evidence above from our article that even fairly far north, it's only during winter that's it may be a problem, it's possible fish and other organisms which naturally live in such latitudes are adapted to storing sufficient vitamin D from the rest of the year for use during winter. I had a brief search but didn't find much about vitamin D in fish (except that it's a good dietary source for humans). Nil Einne (talk) 05:33, 6 May 2012 (UTC)[reply]
An important consideration is that vitamin D can be stored in fat and in the liver for months, so it isn't necessary for an animal to produce or consume adequate amounts of it every month of the year. The general rule is that the fat-soluble vitamins, namely A, D, E and K, can be stored for fairly long periods of time, so it's less important to have a good supply of them constantly, whereas a more regular source needs to the be available of the water-soluble vitamins like C. Vitamin B12 breaks the rule in that it's water soluble, but it can be stored for a long period (400 day half-life in a human liver) anyway. Red Act (talk) 05:41, 6 May 2012 (UTC)[reply]
The fish and seals might migrate. In the case of fish, I wonder how they get enough sunlight, do they come to the surface to "sunbathe" ? StuRat (talk) 06:14, 6 May 2012 (UTC)[reply]

Well, looking into this more, I find this, a study which shows that even the full winter sun promotes no vitamin D synthesis in Boston. Further north, in Edmonton, vitamin D can only be synthesized for six months out of the year. But given its slow loss from the human body, that would seem to be enough. But certainly, there are entire freaking countries further north than Edmonton, but maybe even six months of vitamin D synthesis is more than you need.

Looking for actual information on vitamin D synthesis in fish, I am similarly having trouble finding anything. I did find one old paper explaining that aquarium-reared catfish require dietary vitamin D. Another paper took things a step further, and showed that even when their skin is exposed to sunlight, carp and halibut still require most of their vitamin D from dietary sources. Numerous other papers in the Science Direct database reported the absence of vitamin D synthesis in a variety of fish species.

All of that suggests that even the best natural dietary source of vitamin D we know of is itself getting its vitamin D from diet, but where? Well, maybe plankton. This paper reported that, during summer months, plankton contain high amounts of vitamin D.

Ultimately, it would appear the review we are all looking for is this one , which also argues along the plankton route (yay, one that's actually free to read finally). The argument is made in terms of the ecological pyramid. Basically, since 1000 pounds of plankton needs be consumed at the bottom for one pound of seal to be made at the top (or 10,000 pounds of plankton if you're making a pound of whale), even this seemlingly insignificant organism might much of the oceans' vitamin D through sequestration and concentration as this vitamin makes its way up the food change.

Then again, it's still not clear what the heck plankton itself uses the vitamin for, but it has been suggested to function as a UV filter. Someguy1221 (talk) 06:41, 6 May 2012 (UTC)[reply]

Eh, a teeny bit more since I just realized I still haven't precisely answered the question. Various lines of research indicate that even in extreme northern regions, for at least some portion of the year there is enough sunlight for vitamin D synthesis. Furthermore, since the vitamin D cycle in these regions seem to be plankton→fish→predators, in addition to de novo synthesis in land animals (e.g. humans), even an inefficiently low rate of vitamin D synthesis can potentially provide all the vitamin D for the ecosystem thanks to the effect of the ecological pyramid. Finally, since vitamin D is fat-soluble, it can be efficiently absorbed and concentrated from dietary sources, and stored for long periods of time. Someguy1221 (talk) 09:33, 6 May 2012 (UTC)[reply]
just to clarify in reality it's more like plankton(algae)→plankton→plankton→fish→fish→predator this will allow the fat soluble vitamin to reach high concentrations through biomagnification as Someguy pointed out in his previous post. In fact polar bear and seal livers can have toxic quantities of Vitamin A. See Liver_(food). Staticd (talk) 17:39, 6 May 2012 (UTC)[reply]

Oil near the Great Lakes

Approximately how much oil (petroleum) is drilled from the Great Lakes Basin a year? — Preceding unsigned comment added by 64.229.204.143 (talk) 04:10, 6 May 2012 (UTC)[reply]

On the Canadian side, the annual oil production for the province of Ontario in 2006 was about 150,000 m3 (~900,000 barrels), with a further 31 million m3 being the remaining potential, most in the Great Lakes area [19]. Mikenorton (talk) 07:46, 6 May 2012 (UTC)[reply]
On the US side there is no oil production currently from the Great Lakes area and exploration drilling into the lake beds is currently banned. There was a little oil production in Michigan from wells drilled onshore, but these currently only produce gas with some limited liquids (~7,000 barrels in 2007) [20]. Mikenorton (talk) 08:10, 6 May 2012 (UTC)[reply]

entanglement Logic

does the following logic make sense? If A is true and B is true,(as if entangled) then if A were not true, B would not be true. Or more specifically: because my son and i coexist, if my son never existed I would have never existed either.68.83.98.40 (talk) 05:02, 6 May 2012 (UTC)[reply]

It does make sense if you and your son are entangled. Plasmic Physics (talk) 06:06, 6 May 2012 (UTC)[reply]
If you do not assume that you and your son are entangled but still hold that A therefore B ⇒ Not A, therefore not B, then you are denying the antecedent, a logical fallacy. Someguy1221 (talk) 06:47, 6 May 2012 (UTC)[reply]

The logic is not A therefore B, but A AND B experienced TOGETHER then not A therefore not B. See the difference?68.83.98.40 (talk) 12:38, 6 May 2012 (UTC)[reply]

You may find the article Logical connective to be informative in this sort of discussion. --Jayron32 12:47, 6 May 2012 (UTC)[reply]
I don't think entanglement (which is several concepts in theoretical physics) has anything to do with abstract logic. But if by "entangled" you just mean
A ⇔ B

then indeed

A ⇔ B and ¬A implies ¬B

What percentage of human zygotes end up dying in the womb?

My question is: What percentage of human zygotes end up dying in the womb?

A little background: I remember that years ago back in high school science class I learned that the vast majority of human zygotes die in the womb. A very high percentage was given and I vaguely remember it to be around 80%. Out of random curiosity I tried to find that percentage today and to my shock the answer didn't come easily. A whole range of numbers from 30% to 90% were thrown around in yahoo answers and blog posts with no citation to back them up. I have zero medical training so I'm unable to effectively search through PubMed or Google Scholar because I don't know the right keywords.

Clarifications: I phrased the question as clearly and unambiguously as possible but miscommunications are still possible so I'd like to clarify what I'm not looking for:

  • I'm not looking for data on embryo or fetus. By definition the survival rate for zygotes is lower or equal to the survival rate of embryos which is in turn lower or equal to the survival rate of fetuses. I'm only interested in survival rate for zygotes, which is the "worst case rate" that factors in the survival rate of embryos and fetuses.
  • I'm not looking for miscarriage data. Multiple zygotes can die in a woman and she can still have her normal period within the same month; there is no miscarriage in this case.Anonymous.translator (talk) 12:45, 6 May 2012 (UTC)[reply]
A zygote that dies very early on will usually not leave any signs that conception ever took place, which makes it very difficult to get accurate numbers on how often it happens. That's probably why you couldn't find an definite answer. --Tango (talk) 14:23, 6 May 2012 (UTC)[reply]
Like Tango said, this is very difficult to determine, however, this article could be interesting in this context. It suggests that the maximum chance of (clinically recognized) pregnancy (30-40%) is (mainly) determined by the rate of preclinical pregnancy loss. That is to say, probably most of the 60-70% of women who fail to conceive in optimal conditions experience early pregnancy loss. Combined with the rate of clinical spontaneous abortion of 8%, one could estimate that (1-0.4*0.92=0.63; 1-0.3*0.92=0.72) 63-72% of fertilized eggs die in the womb, but this assumes a 100% conception rate in optimal conditions, which is probably lower. - Lindert (talk) 14:46, 6 May 2012 (UTC)[reply]

Virtual work

Are there any situations, even very contrived situations, where constraint forces do virtual work? 65.92.6.118 (talk) 15:03, 6 May 2012 (UTC)[reply]

Do you mean like Gold farming and Virtual economy? Unique Ubiquitous (talk) 17:54, 6 May 2012 (UTC)[reply]
I suspect the original poster means virtual work. -- Finlay McWalterTalk 18:25, 6 May 2012 (UTC)[reply]

Is it safe to keep batteries in a tin can?

I keep my batteries in a red tin can and a brown tin can. I put the dead ones in the red can and the ones that I haven't used yet in the brown can. After a while this chemical smell started come around. So is this safe? Oh and can I recycle batteries? Matthew Goldsmith 18:58, 6 May 2012 (UTC)